hidrostatica lista de exercícios

Propaganda
EXERCÍCIOS DE REFORÇO
1
Hidrostática
1) Um corpo de massa 600 gramas ocupa volume de 200 cm³. Calcule a densidade desse corpo em g/cm³, kg/L e kg/m³.
2) Complete as equivalências entre as unidades:
a) 1 g/cm³ =…..kg/m³ = ………kg/L
b) 1 kg/m3 = ......g/cm3 = …….kg/L
3) Sabendo que a massa específica do alumínio é 2,7 g/cm³, calcule:
a) a massa de um corpo maciço de alumínio cujo volume é 30 cm3;
b) o volume de um corpo maciço de alumínio cuja massa é 2,16 kg.
4)Uma pessoa comprime um percevejo contra uma mesa de madeira, exercendo uma força de 20 N. Sabendo que a ponta
do percevejo tem área 0,10 mm2,calcule, em N/m2,a pressão exercida pela ponta do percevejo.
5) Um corpo de cobre tem a forma de uma casca esférica de raio interno R, = 4,00 cm e raio externo R2= 10,0 cm (a
parte interna é oca). Calcule a densidade do corpo sabendo que a massa específica do cobre é 8,92 g/cm 3.Dado: volume
da esfera =
4
 R³
3
6) Numa região em que g = 10 m/s²,um homem de massa 60 kg está de pé, usando
sapatos, exercendo sobre os pontos de contato com o solo uma pressão média de
2,0 . 104 Pa. Calcule a área de contato entre os sapatos e o solo:
a) em m2
b) em cm2
7) Na figura abaixo representamos um recipiente contendo um líquido de
densidade d = 2,4 . 103kg/m3. Sabendo que g = 10 m/s2, h = 3,0 m e que a pressão
no ponto A é 1,2 . 105Pa, calcule a pressão no ponto B.
8) Numa região onde a pressão atmosférica é 1,0. 105N/m2e g = 10 m/s2, uma
caixa cúbica de aresta 2,0 m encontra-se no fundo de um lago, estando a face
superior da caixa a uma profundidade h = 40 m. Supondo a densidade da água
igual a 1,0 .103 kg/m3, calcule:
a) a pressão na face superior da caixa;
b) a força exercida pela água na face superior da caixa.
9) Numa região em que g = 10 m/s2 e a pressão atmosférica é Patm= 1,0 .105Pa temos óleo e água
em equilíbrio dentro de um recipiente, como mostra a figura. Sabendo que a densidade do óleo é
9,0. 102kg/m3 e a densidade da água é 1,0. 103kg/m3.calcule:
a) a pressão no ponto X;
b) a pressão no ponto Y.
10) Num tubo em U estão em equilíbrio dois líquidos imiscíveis (água e mercúrio), como mostra a
figura. Sabendo que a densidade da água é 1,0g/cm³ e a densidade do mercúrio é 13,6 g/cm³,calcule o desnível h entre as
superfícies livres dos dois líquidos.
2
11) No elevador hidráulico representado na figura, os êmbolos E1 e E2, de pesos desprezíveis, têm áreas respectivamente
iguais a 80 cm2 e 800 cm2. Qual é a intensidade da força F necessária para sustentar um automóvel de peso P = 12000 N?
12) (U. E. Londrina-PR) Para medir a pressão p exercida por um gás, contido num recipiente, utilizou-se um manômetro
de mercúrio, obtendo-se os valores indicados na figura abaixo.
A pressão atmosférica local medida por um barômetro indicava 750 mmHg. O valor de p, em mmHg, vale:
a) 150
b) 170
c) 750
d) 900
e) 940
13) Quando você bebe um líquido usando canudinho, por que o líquido sobe pelo canudo?
14) Um bloco de volume 2,0 . 10-3 m3 e densidade 3,0 . 103 kg/m3 está totalmente submerso na água, cuja densidade é 1,0
. 103kg/m3,e preso por um fio a um dinamômetro, como mostra a figura. Dado g = 10 m/s2,calcule:
a) a massa do bloco;
b) o empuxo sobre o bloco;
c) a marcação do dinamômetro (peso aparente).
15) Na figura abaixo representamos um corpo esférico flutuando em um líquido de densidade 0,80
g/cm3 com
4
de seu volume submerso. Qual é a densidade do corpo?
5
3
16) A densidade do aço é maior que a da água. No entanto, um navio cujo casco é de aço consegue flutuar na água.
Como isso é possível?
Vestibular
1) (Fuvest-SP) Uma chapa de cobre de 2 m², utilizada em um coletor de energia solar, é pintada com tinta preta cuja
massa específica, após a secagem, é 1,7 g/cm³. A espessura da camada é da ordem de 5 m (micrometro). Qual é a massa
de tinta seca existente sobre a chapa?
2) (UF-RS) Um gás está contido em um recipiente cúbico, tendo cada face área igual a 2,0 m².As moléculas do gás
bombardeiam continuamente as faces do recipiente exercendo sobre elas uma pressão média de 5,0 .10³ Pa. Qual o
módulo da força média exercida pelo gás sobre cada face?
a) 1,0.104N
b) 7,5. 103N
c) 5,0.103N
d) 2,5.103N
e) 1,0.103N
3) (U. F.Uberaba-MG) Considere os recipientes A, B e C da figura, cujas áreas das paredes do fundo são iguais. Os
recipientes contêm o mesmo líquido homogêneo em equilíbrio, sendo que em todos eles o nível livre do líquido atinge a
altura h. Em relação às pressões PA, PB e Pc exercidas pelo líquido nas paredes do fundo dos recipientes A, B e C,
respectivamente, é correto que:
a) PA> PB> Pc
d) PA= PB> Pc
b) PA< PB< Pc
e) PA< PB= Pc
c) PA= PB= Pc
4) (UF-PR) Em um manômetro de tubo aberto, a diferença de alturas entre as colunas de mercúrio é 38 cm. Sendo a
experiência realizada ao nível do mar, pode-se afirmar que a pressão do gás é:
a) 0,50 atm
b) 1,0 atm
c) 1,5 atm
d) 1,9 atm
e) 3,8 atm
5) (U. E. Londrina-PR) Na prensa hidráulica representada abaixo, os diâmetros dos
êmbolos são d1 e d2,tais que d1 = 2 d2.
A relação
F1
entre as intensidades das forças exercidas nos dois êmbolos, quando
F2
situados no mesmo nível, vale:
a) 4
b) 2
c) 1
d)
1
2
e)
1
4
6) (UF-CE) Um mergulhador pode suportar uma pressão máxima de 10 vezes a pressão atmosférica Po, tomando g = 10
m/s² e Po= 1,0 .105 N/m2, calcule a que profundidade máxima, em metros, pode o mergulhador descer abaixo da
superfície de um lago, onde a densidade da água é 1,0. 103kglm3.
ENEM
7) Pelas normas vigentes, o litro do álcool hidratado que abastece os veículos deve ser constituído de 96% de álcool puro
e 4% de água (em volume). As densidades desses componentes são dadas na tabela.
Densidade (g/L) 1000 800
Substância
Água Álcool
4
Um técnico de um órgão de defesa do consumidor inspecionou cinco postos suspeitos de venderem álcool hidratado fora
das normas. Colheu uma amostra do produto em cada posto, mediu a densidade de cada uma, obtendo:
Posto
I
II
III IV V
Densidade do combustível (g/L) 822 820 815 808 805
A partir desses dados, o técnico pôde concluir que estavam com o combustível adequado somente os postos:
a) I e II.
b) I e III.
c) II e IV.
d) III e V.
e) IV e V.
8)A adaptação dos integrantes da seleção brasileira de futebol à altitude de La Paz foi muito comentada em 1995, por
ocasião de um torneio, como pode ser lido no texto abaixo:
A seleção brasileira embarcará hoje para La Paz, capital da Bolívia, situada a 3700 metros de altitude, onde disputará o
torneio Interamérica. A adaptação deverá ocorrer em um prazo de 10 dias, aproximadamente. O organismo humano, em
altitudes elevadas, necessita desse tempo para se adaptar, evitando-se, assim, risco de um colapso circulatório.
(Adaptado da revista Placar, edição fev. 1995.)
A adaptação da equipe foi necessária principalmente porque a atmosfera de La Paz, quando comparada à das cidades
brasileiras, apresenta:
a) menor pressão e menor concentração de oxigênio.
b) maior pressão e maior quantidade de oxigênio.
c) maior pressão e maior concentração de gás carbônico.
d) menor pressão e maior temperatura.
e) maior pressão e menor temperatura.
Calor e Temperatura
1) Assinale a alternativa que define corretamente calor.
a) Trata-se de um sinônimo de temperatura em um sistema.
b) É uma forma de energia contida nos sistemas.
c) É uma energia em trânsito, de um sistema a outro, devido à diferença de temperatura entre eles.
d) É uma forma de energia superabundante nos corpos quentes.
e) É uma forma de energia em trânsito, do corpo mais frio para o corpo mais quente.
2) Quando podemos dizer que dois corpos estão em equilíbrio térmico?
3) Faça as seguintes transformações:
a) 30°C para °F
b) 100°F para °C
c) 0°F para °C
d) -30°F para K
4)Há uma temperatura que tem o mesmo valor nas escalas Celsius e Fahrenheit. Qual é essa temperatura?
5) Quando um termômetro graduado na escala Celsius sofrer uma variação de 40 graus em sua temperatura, qual será a
correspondente variação de temperatura para um termômetro graduado na escala Fahrenheit?
6)O gráfico abaixo relaciona a temperatura e a altura da coluna líquida de um
termômetro.
a) Escreva a equação que relaciona  e h.
b) Qual é a temperatura correspondente a uma altura de 24 cm?
7) Certa vez um jornal publicou a seguinte notícia: "Ontem os termômetros registraram
a temperatura mais quente dos últimos dez anos." Nessa frase há um erro conceitual.
Qual?
8) Uma barra de cobre, cujo coeficiente de dilatação linear é 17x10-6°C-1,tem comprimento de 200,0 cm à temperatura de
50°C. Calcule o comprimento dessa barra à temperatura de 450°C.
5
9) A figura representa a variação do comprimento de uma determinada barra homogênea. Determine valor do coeficiente
de dilatação linear do material de que é constituída a barra.
10) Na figura está representada uma lâmina bimetálica. O coeficiente de dilatação do metal
da parte superior (1) é o dobro do coeficiente do metal da parte inferior (2). À temperatura
ambiente, a lâmina é horizontal. Se a temperatura for aumentada 150°C, a lâmina:
a) continuará horizontal.
b) curvará para baixo.
c) curvará para cima.
d) curvará para a direita.
e) curvará para a esquerda.
11) Duas barras, uma de aço e outra de zinco, apresentam, respectivamente, comprimentos de 220,4 cm e 220,0 cm a
20°C. Sabe-se que os coeficientes de dilatação linear do aço e do zinco são, respectivamente, 12 x10-6°C-1 e 27x10-6°C-1
A que temperatura essas barras terão o mesmo comprimento?
12) Uma chapa quadrada tem lado de 80 cm a 20°C. Sabendo que o material de que é feita a chapa tem coeficiente de
dilatação linear 1,5 x 10-5°C-1 calcule:
a) o coeficiente de dilatação superficial da chapa;
b) a área da chapa a 60°C.
13) Um corpo tem forma de cubo, cuja aresta mede 10 cm a 30°C. O material de que é feito o corpo tem coeficiente de
dilatação linear 12 x 10-6°C-1. Calcule:
a) o coeficiente de dilatação volumétrica do corpo;
b) o volume do corpo a 50°C.
14) Um recipiente de vidro cujo volume interno é 1600 cm³ está cheio de mercúrio, à temperatura de 20°C. Os
coeficientes de dilatação volumétrica do vidro e do mercúrio são, respectivamente, Yv= 2,7 x 10 -5°C-1 e YM= 1,8 x 105
°C-1. Se o conjunto for aquecido à temperatura de 120°C, qual é o volume do mercúrio que transborda?
15) Um recipiente contém certa massa de água na temperatura inicial de 2 °C e na pressão normal;quando é aquecido,
sofre uma variação de temperatura de 3°C. Pode-se afirmar que, nesse caso, o volume de água:
a) diminui e após aumenta.
b) aumenta e após diminui.
c) diminui.
d) aumenta.
e) permanece constante.
16) Um menino encheu de água uma garrafa de vidro e a seguir colocou-a no congelador. Depois de algum tempo a
garrafa estourou. Por quê?
17) Um bloco de vidro de massa m = 300 gramas está inicialmente à temperatura Ti = 25°C. Sabendo que o calor
específico do vidro é c = 0,20 cal/g°C, calcule a quantidade de calor necessária para elevar a temperatura do bloco até Tf
= 40°C.
18) Ao receber a quantidade de calor Q =600cal, um corpo de massa 50 gramas tem sua temperatura aumentada em
20°C. Qual é a capacidade térmica do corpo? E o calor especifico?
19)Um corpo de capacidade térmica C = 25 J/ºC está inicialmente à temperatura de 30°C. Se esse corpo receber uma
quantidade de calor Q = 200 J, qual será a sua temperatura final? (Suponha que não haja mudança de estado de
agregação.)
6
20) Um corpo de capacidade térmica C = 150 cal/ºC está inicialmente à temperatura de 60°C. Se esse corpo perder uma
quantidade de calor de 1200 cal, qual será sua temperatura final? (Suponha que não haja mudança de estado.)
21) O gráfico abaixo nos mostra a variação da temperatura de um corpo em função do tempo.
Sabendo que esse corpo está perdendo calor à razão de 10 cal/s, calcule:
a) a quantidade de calor perdida pelo corpo em 1 minuto;
b) a capacidade térmica do corpo.
22) Um bloco de alumínio tem massa 500 g. Sabendo que o calor específico do alumínio é 0,22 cal/gºC, calcule a
capacidade térmica desse bloco.
23) Um objeto é feito de duas partes: uma parte de alumínio, cuja massa é 1000 gramas, e uma parte de ferro, cuja massa
é 600 gramas. Sabendo que os calores específicos do alumínio e do ferro são, respectivamente, c Al = 0,22 cal/gºC e cFe =
0,11 cal/gºC, calcule a capacidade térmica desse objeto.
24) Um bloco de cobre tem massa 2000 gramas e temperatura inicial 25°C. Sabendo que o calor específico do cobre é
0,093 cal/gºC, calcule a quantidade de calor necessária para elevar a temperatura do bloco para 65°C.
25) Um recipiente contém 3,0 kg de água inicialmente à temperatura de 60°C. Depois de algum tempo a temperatura da
água diminui para 40 ºC. Sabendo que o calor específico da água é 1,0 cal/g°C,qual é a quantidade de calor perdida pela
água nesse intervalo de tempo?
26) Uma fonte de potência P = 200 W é utilizada para aquecer um corpo feito de um único
material. O gráfico abaixo nos dá a temperatura do corpo em função do tempo. Sabendo que a
massa do corpo é m = 2,0 kg e adotando 1 cal = 4,0 J, calcule o calor específico do material de
que é feito o corpo.
27) Em um calorímetro de capacidade térmica desprezível há 660 gramas de água a 20°C.
Jogando dentro do calorímetro uma bola de alumínio de massa 300 gramas à temperatura de
240°C, qual será a temperatura do conjunto no equilíbrio térmico?
São dados: calor específico da água = 1,0 cal/gºC; calor específico do alumínio = 0,22 cal/gºC.
28) Um calorímetro de capacidade térmica 100 cal/ºC contém 500 gramas de água à temperatura de 30°C. Um corpo de
cobre cuja massa é 200 gramas e cuja temperatura é 300°C é jogado dentro do calorímetro. Calcule a temperatura de
equilíbrio. São dados: calor específico da água = 1,0 cal/gºC; calor específico do cobre = 0,093 cal/gºC.
29) Em um calorímetro cuja capacidade térmica é 50 cal/ºC há 400 gramas de água à temperatura de 10°C. Um objeto
metálico de massa 150 gramas e temperatura 50 °c é lançado dentro do calorímetro. Sabendo que a temperatura final da
mistura é 30°C, calcule o calor específico do metal. (O calor específico da água é 1,0 cal/gºC.)
30) Em um calorímetro de capacidade térmica desprezível há 330 gramas de água a 25°C. Uma pequena esfera de
alumínio de massa 100 gramas é jogada dentro do calorímetro. Sabendo que a temperatura de equilíbrio é 27°C, calcule a
temperatura da esfera de alumínio no momento em que foi lançada no calorímetro. São dados: calor específico da água =
1,0 cal/gºC;calor específico do alumínio = 0,22 cal/gºC.
31) Uma dona de casa em uma cidade litorânea, para seguir a receita de um bolo, precisa de uma xícara de água a 50°C.
Infelizmente, embora a cozinha seja bem aparelhada, ela não tem termômetro. Como pode a dona de casa resolver o
problema? (Você pode propor qualquer procedimento correto, desde que não envolva termômetro.)
32) O ponto de fusão do chumbo é 327°C e seu calor de fusão é 5,9 cal/g. Se tivermos um bloco de chumbo de massa
100 g a 327°C, qual será a quantidade de calor necessária para fundir esse bloco?
31)Consideremos um cubo de gelo de massa 400 gramas à temperatura de -15ºC. Calcule a quantidade de calor
necessária para transformar esse gelo em vapor d'água a 150°C. São dados: calor específico do gelo = 0,50 cal/g °C;
7
calor específico da água líquida = 1,00 cal/g °C; calor específico do vapor d'água = 0,48 cal/g °C; calor de fusão do gelo
= 80 cal/g; calor de vaporização da água = 540 cal/g.
33) No diagrama representamos a temperatura de um corpo de massa 100 g (feito de um
único material) em função da quantidade de calor recebido. Inicialmente o corpo está no
estado sólido à temperatura de 20°C. Determine:
a) as temperaturas de fusão e ebulição desse material;
b) os calores de fusão e vaporização desse material;
c) os calores específicos desse material nos estados sólido, líquido e de vapor.
34) Em um calorímetro de capacidade térmica desprezível, há 500 gramas de água a 60°C. Se colocarmos dentro dele
200 gramas de gelo a O°C, qual será a temperatura de equilíbrio? São dados: calor específico da água líquida = 1,0 cal/g
°C; calor de fusão do gelo = 80 cal/g.
35) Sob pressão normal, pode-se atravessar um bloco de gelo a uma temperatura um pouco inferior a O°C, usando-se um
arame com dois corpos "pesados" presos nas extremidades, como indica a figura. A explicação para isso é que:
a) a pressão exercida pelo arame sobre o gelo abaixa seu ponto de ebulição.
b) o gelo já cortado pelo arame, devido à baixa temperatura, funde-se novamente.
c) a pressão exercida pelo arame sobre o gelo aumenta seu ponto de fusão, mantendo a barra sempre sólida.
d) o arame estando naturalmente mais aquecido funde o gelo; esse calor, uma vez perdido para a atmosfera, deixa a barra
novamente sólida.
e) a pressão exercida pelo arame abaixa o ponto de fusão do gelo que se funde. O arame penetra no bloco e com o "alívio
da pressão" o bloco se solidifica novamente
36) No verão usamos aparelhos de ar condicionado para refrigerar o ambiente. Onde ele deve ser instalado: na parte
superior ou inferior da parede?(Explique)
37) Ao contato da mão e à temperatura ambiente de 25°C, o mármore parece mais frio que a madeira porque:
a) a madeira está sempre acima da temperatura ambiente.
b) o mármore não alcança a temperatura ambiente.
c) o calor da mão se escoa rapidamente para o mármore, em virtude da grande condutibilidade térmica desse material.
d) a madeira possui mais condutibilidade térmica do que o mármore.
e) a capacidade térmica do mármore tem valores muito diferentes para pequenas variações de temperatura.
38)Por que a serragem é melhor isolante que a madeira?
39) Em dias frios os pássaros eriçam suas penas e com isso conseguem perder menor calor para o ambiente. Por quê?
40) Num planeta completamente desprovido de fluidos, apenas pode ocorrer propagação de calor por:
a) convecção e condução
b) convecção e irradiação
c) condução e irradiação
d) irradiação.
e) convecção
41) Nas praias notamos que, em geral, durante o dia o vento vem do mar para a praia e à noite o vento vai da praia para o
mar. Por quê?
Vestibular
1) (Fatec-SP) Um termômetro de mercúrio foi calibrado de tal modo que a temperatura de 0 °C correspondesse a 4 cm de
altura da coluna de mercúrio; a temperatura de 100°C correspondesse a 8 cm de altura da coluna de mercúrio. A função
termométrica que relaciona a temperatura  (em ºC) e a altura h (em cm) é:
8
a)  = 25h - 4
b)  = 100 (h - 4)
c)  = 25 (h - 1)
d)  = 25 (h - 4)
e)  = 4 (25 - h)
2) (Vunesp-SP) Uma barra de latão de 1,0 m sofre um acréscimo de comprimento de 1,0 mm quando sua temperatura se
eleva de 50°C. A partir desses dados, pode-se concluir que o coeficiente de dilatação linear do latão, em °C-I, é de:
a) 8,0. 10-5
b) 6,0. 10-5
c) 4,0. 10-5
d) 2,0. 10-5
e) 1,0. 10-5
3)(Unifor-CE) O gráfico representa a variação dos comprimentos, em função da temperatura, de duas barras metálicas
Me N. Podemos afirmar que:
a) os coeficientes de dilatação das barras M e N são iguais.
b) a barra M tem maior coeficiente de dilatação do que a barra N.
c) o coeficiente de dilatação da barra M é menor do que o da barra N.
d) as barras M e N têm sempre o mesmo comprimento. L
4) (UF-RS) O gráfico mostra o acréscimo relativo no comprimento de uma barra
metálica uniforme, à medida que sua temperatura aumenta. O coeficiente de
dilatação linear do metal é igual a:
a) 1,5. 104 °C-1
d) 1,5. 10-5 °C-1
-2
-1
b) 1,5.10 °C
e) 1,5 .10-7 °C-1
-4
-1
c) 1,5. 10 °C
5) (UE-CE) A figura mostra uma pequena bola em repouso sobre uma barra horizontal, sustentada por dois fios de metais
diferentes, (1) e (2), comprimentos desiguais, LI e L2 a 0°C, respectivamente.
Sendo 1 e 2 os respectivos coeficientes de dilatação dos fios (1) e (2), qual das
relações a seguir representa a condição para que a bola continue equilibrada sobre a
barra, ao variar a temperatura?
a) 1 = 2
b) 1 . L1 = 2 L2
c) 1 . L2 = 2 L1
d) L2 L1 = 2 1
6)(UF-PR) Para aquecer 500 g de certa substância de 20°C a 70°C, foram necessárias 4000 calorias. A capacidade
térmica e o calor específico dessa substância são, respectivamente:
a) 8 cal/ºC e 0,08 cal/gºC
b) 80 cal/ºC e 0,16 cal/gºC
c) 90 cal/ºC e 0,09 cal/ºC
d) 95 cal/ºC e 0,15 cal/gºC
e) 120 cal/ºC e 0,12 cal/gºC
7) (Fuvest-SP) O gráfico representa a variação da temperatura de um corpo sólido,
em função do tempo, ao ser aquecido por uma fonte que libera energia à razão de 150
cal/min. Como a massa do corpo é de 100 g, o seu calor específico, em cal/gºC,será
de:
a) 0,75
b) 3,75
c) 7,50
d) 0,80
e) 1,50
8) (Fuvest-SP) Um aquecedor de água, que utiliza energia solar, absorve num dia ensolarado uma potência de 2 000 W.
Para aquecer 100 litros de água, de 15°C a 40°C, nesse aquecedor, desprezando-se as perdas, serão necessários
aproximadamente:
a) 10 minutos.
b) 20 minutos
c) 40 minutos.
d) 80 minutos
e) 160minutos.
Dado: calor específico da água = 4J .g-1 .oC-1.
9
9) (Efoa-MG) Um corpo de alumínio com massa de 10 g e temperatura de 80°C é submerso em água, com massa de 10 g
e temperatura de 20°C. Considere só as trocas de calor entre o alumínio e a água; a temperatura de equilíbrio térmico é
de:
a) 50 °C
b) 30°C
c) 40 °C
d) 60°C
e) 70 °C
Dado:calor específico do alumínio:c = 0,20 cal/gOC.
10) (PUC-RJ) Quando misturamos 100 g de gelo a O°C com 900 g de água a 20°C em um recipiente termicamente
isolado e de capacidade térmica desprezível, a temperatura final de equilíbrio é igual a:
a) 4°C
b) 10°C
c) 14°C
d) 15°C
e) 20°C
Dados: calor de fusão do gelo = 80 cal/g; calor específico da água = 1,0 cal/gºC.
11) (UF-PB) Misturam-se 60 gramas de água a 20°C com 80 g de gelo a 0ºC. Supondo que só há trocas de calor entre a
água e o gelo, calcule a temperatura final do líquido. Dados: calor de fusão do gelo = 80 cal/g;calor específico da água =
1,0 cal/g . ºC
12) (Acafe-Se) Em Camboriú, a pressão atmosférica equivale a 76 cmHg e a água ferve a 100°C. Em relação a
Camboriú, no Pico da Neblina, ponto culminante do Brasil, a pressão atmosférica e o ponto de ebulição da água são,
respectivamente:
a) menor e menor.
b) maior e maior
c) maior e menor.
d) menor e maior.
. e) igual e igual.
13) (Fuvest-SP) Têm-se dois corpos, com a mesma quantidade de água, um aluminizado A e outro negro N, que ficam
expostos ao Sol durante uma hora. Sendo inicialmente as temperaturas iguais, é mais provável que ocorra o seguinte:
a) Ao fim de uma hora não se pode dizer qual temperatura é maior.
b) As temperaturas são sempre iguais em qualquer instante.
c) Após uma hora a temperatura N é maior que a de A.
d) De início, a temperatura de A decresce (devido à reflexão) e a de N aumenta.
e) As temperaturas de N e de A decrescem (devido à evaporação) e depois crescem.
14)(FCMSC-SP) Os iglus, embora feitos de gelo, possibilitam aos esquimós residir neles porque:
a) o calor específico do gelo é maior que o da água.
b) o calor específico do gelo é extraordinariamente pequeno, comparado ao da água.
c) a capacidade térmica do gelo é muito grande.
d) o gelo não é um bom condutor de calor.
e) a temperatura externa é igual à interna.
15) (F. M. ABC-SP) Atualmente, os diversos meios de comunicação vêm alertando a população para o perigo que a
Terra começa a enfrentar, o chamado "efeito estufa". Tal efeito é devido ao excesso de gás carbônico presente na
atmosfera, provocado pelos poluentes, pelos quais o homem é responsável direto. O aumento de temperatura provocado
pelo fenômeno deve-se ao fato de que:
a) a atmosfera é transparente à energia radiante e opaca para as ondas de calor.
b) a atmosfera é opaca à energia radiante e transparente para as ondas de calor.
c) a atmosfera é transparente tanto para a energia radiante como para as ondas de calor.
d) a atmosfera é opaca tanto para a energia radiante como para as ondas de calor.
e) a atmosfera funciona como um refletor para a energia radiante e como meio absorvente para a energia térmica.
ENEM
1) Uma garrafa de vidro e uma lata de alumínio, cada uma contendo 330 mL de refrigerante, são mantidas em um
refrigerador pelo mesmo longo período de tempo. Ao retirá-las do refrigerador com as mãos desprotegidas, tem-se a
sensação de que a lata está mais fria que a garrafa. É correto afirmar que:
a) a lata está realmente mais fria, pois a capacidade calorífica da garrafa é maior que a da lata.
b) a lata está de fato menos fria que a garrafa, pois o vidro possui condutividade menor que o alumínio.
c) a garrafa e a lata estão à mesma temperatura, possuem a mesma condutividade térmica, e a sensação deve-se à
diferença nos calores específicos.
10
d) a garrafa e a lata estão à mesma temperatura, e a sensação é devida ao fato de a condutividade térmica do alumínio ser
maior que a do vidro.
e) a garrafa e a lata estão à mesma temperatura, e a sensação é devida ao fato de a condutividade térmica do vidro ser
maior que a do alumínio.
2) Ainda hoje, é muito comum as pessoas utilizarem vasilhames de barro (moringas ou potes de cerâmica não esmaltada)
para conservar água a uma temperatura menor do que a do ambiente. Isso ocorre porque:
a) o barro isola a água do ambiente, mantendo-a sempre a uma temperatura menor que a dele, como se fosse isopor.
b) o barro tem poder de "gelar" a água pela sua composição química. Na reação, a água perde calor.
c) o barro é poroso, permitindo que a água passe através dele. Parte dessa água evapora, tomando calor da moringa e do
restante da água, que são assim resfriadas.
d) o barro é poroso, permitindo que a água se deposite na parte de fora da moringa. A água de fora sempre está a uma
temperatura maior que a de dentro.
e) a moringa é uma espécie de geladeira natural, liberando substâncias higroscópicas que diminuem naturalmente a
temperatura da água.
3) A tabela a seguir registra a pressão atmosférica em diferentes altitudes, e o gráfico relaciona a pressão de vapor da
água em função da temperatura:
Altitude (km)
0
1
2
4
6
8
10
Pressão atmosférica (mmHg) 760 600 480 300 170 120 100
Um líquido, num frasco aberto, entra em ebulição a partir do
momento em que a sua pressão de vapor se iguala à pressão
atmosférica. Assinale a opção correta, considerando a tabela,
o gráfico e os dados apresentados sobre as seguintes cidades:
Natal (RN)
nível do mar
Campos do Jordão (SP) altitude 1628 m
Pico da Neblina (AM)
altitude 3014 m
A temperatura de ebulição será:
a) maior em Campos do Jordão.
b) menor em Natal.c) menor no Pico da Neblina.
d) igual em Campos do Jordão e Natal.
e) não dependerá da altitude.
4) A gasolina é vendida por litro, mas em sua utilização como combustível a massa é o que importa. Um aumento da
temperatura do ambiente leva a um aumento no volume da gasolina. Para diminuir os efeitos práticos dessa variação, os
tanques dos postos de gasolina são subterrâneos. Se os tanques não fossem subterrâneos:
I. Você levaria vantagem ao abastecer o carro na hora mais quente do dia, pois estaria comprando mais massa por litro de
combustível.
II.Abastecendo com a temperatura mais baixa, você estaria comprando mais massa de combustível para cada litro.
III. Se a gasolina fosse vendida por quilograma em vez de por litro, o problema comercial decorrente da dilatação da
gasolina estaria resolvido. Dessas considerações, somente:
a) I é correta.
b) II é correta.
c) III é correta.
d) I e II são corretas. e) II e III são corretas.
5) A panela de pressão permite que os alimentos sejam cozidos em água muito mais rapidamente do que em panelas
convencionais. Sua tampa possui uma borracha de vedação que não deixa o vapor escapar, a não ser através de um
orifício central sobre o qual assenta um peso que controla a pressão.Quando em uso, desenvolve-se uma pressão elevada
no seu interior. Para a sua operação segura, é necessário observar a limpeza do orifício central e a existência de uma
válvula de segurança, normalmente situada na tampa. O esquema da panela de pressão e um diagrama de fases da água
são apresentados abaixo:
11
A vantagem do uso da panela de pressão é a rapidez para o cozimento de alimentos e isso se deve:
a) à pressão no seu interior,que é igual à pressão externa.
b) à temperatura de seu interior,que está acima da temperatura de ebulição da água no local.
c) à quantidade de calor adicional que é transferida à panela.
d) à quantidade de vapor que está sendo liberada pela válvula.
e) à espessura da sua parede, que é maior que a das panelas comuns.
6) Se, por economia, abaixarmos o fogo sob uma panela de pressão logo que se inicia a saída de vapor pela válvula, de
forma simplesmente a manter a fervura, o tempo de cozimento:
a) será maior,porque a panela"esfria".
b) será menor, pois diminui a perda de água.
c) será maior, pois a pressão diminui.
d) será maior, pois a evaporação diminui.
e) não será alterado, pois a temperatura não varia.
7) A refrigeração e o congelamento de alimentos são responsáveis por uma parte significativa do consumo de energia
elétrica numa residência típica.
Para diminuir as perdas térmicas de uma geladeira, podem ser tomados alguns cuidados operacionais:
I. Distribuir os alimentos nas prateleiras deixando espaços vazios entre eles, para que ocorra a circulação do ar frio para
baixo e do quente para cima.
II.Manter as paredes do congelador com camada bem espessa de gelo, para que o aumento da massa de gelo aumente a
troca de calor no congelador.
III.Limpar o radiador ("grade" na parte de trás) periodicamente, para que a gordura e a poeira que nele se depositam não
reduzam a transferência de calor para o ambiente.
Para uma geladeira tradicional é correto indicar, apenas:
a) a operação I.
b) a operação II.
c) as operações I e II. d) as operações I e III. e) as operações IIe III.
8) O resultado da conversão direta de energia solar é uma das várias formas de energia alternativa de que se dispõe. O
aquecimento solar é obtido por uma placa escura coberta por vidro, pela qual passa um tubo contendo água. A água
circula, conforme mostra o esquema abaixo:
São feitas as seguintes afirmações quanto aos materiais utilizados no aquecedor solar:
I. O reservatório de água quente deve ser metálico para conduzir melhor o calor.
II.A cobertura de vidro tem como função reter melhor o calor, de forma semelhante ao que ocorre em uma estufa.
III.A placa utilizada é escura para absorver melhor a energia radiante do Sol, aquecendo a água com maior eficiência.
Dentre as afirmações acima, pode-se dizer que apenas está(ão) correta(s):
a) I
b) I e II
c) II
d) I e III
e) II e III
12
9) Numa área de praia, a brisa marítima é uma conseqüência da diferença no tempo de aquecimento do solo e da água,
apesar de ambos estarem submetidos às mesmas condições de irradiação solar. No local (solo) que se aquece mais
rapidamente,o ar fica mais quente e sobe, deixando uma área de baixa pressão, provocando o deslocamento do ar da
superfície que está mais fria (mar). À noite, ocorre um processo inverso ao que se verifica durante o dia
Como a água leva mais tempo para esquentar (de dia), mas também leva mais tempo para esfriar (à noite), o fenômeno
noturno (brisa terrestre) pode ser explicado da seguinte maneira:
a) O ar que está sobre a água se aquece mais;ao subir, deixa uma área de baixa pressão,causando um deslocamento de ar
do continente para o mar.
b) O ar mais quente desce e se desloca do continente para a água, a qual não conseguiu reter calor durante o dia.
c) O ar que está sobre o mar se esfria e dissolve-se na água; forma-se, assim, um centro de baixa pressão, que atrai o ar
quente do continente.
d) O ar que está sobre a água se esfria, criando um centro de alta pressão que atrai massas de ar continental.
e) O ar sobre o solo, mais quente, é deslocado para o mar, equilibrando a baixa temperatura do ar que está sobre o mar.
Gases
1)Determinada quantidade de gás ideal ocupa volume de 12 litros à pressão de 1,5 atm. Se o gás sofrer uma expansão
isotérmica, passando a ocupar um volume de 18 litros, qual será sua nova pressão?
2) Na figura ao lado representamos a isoterma correspondente à transformação de um gás ideal.
Determine os valores dos volumes VI e V2
3) Um cilindro, de raio interno R e contendo ar, é provido de um pistão de massa m que pode
deslizar livremente. O sistema está inicialmente em equilíbrio à temperatura de 300 K e a altura h vale 9,0 x 10-2 m. Se o
ar for aquecido até atingir um novo estado de equilíbrio à temperatura de 400 K, o novo valor de h será .... metros:
a) 39,5 x 10-2
b) 12,0 x 10-2 c) 7,00 x 10-2
-2
d) 4,00 x 10
e) 1,58 x 10-2
4) Determinada massa de gás ideal ocupa um volume de 60 litros sob pressão de 2,0 atm, à temperatura de 77 ºC. Se a
temperatura do gás for aumentada mantendo-se a pressão constante, qual será a temperatura do gás quando o volume for
240 litros?
5) Certa massa de gás ideal está inicialmente à temperatura de 400 K e pressão de 4,0 atm. Mantendo-se o volume
constante, a temperatura é reduzida para 320 K. Qual será o novo valor da pressão?
6) Sob pressão de 6,0 atm, certa massa de gás ideal ocupa um volume de 400 litros à temperatura de 300 K. O gás é então
aquecido de modo que sua temperatura passa para 500 K e sua pressão para 8,0 atm. Qual será o novo volume do gás?
7) Consideremos uma amostra de CO2 cuja massa é m = 88 gramas. Calcule o número de mols de moléculas da amostra
sabendo que:
massa molar do carbono = Me = 12 gramas/mol
massa molar do oxigênio = Mo = 16 gramas/mol
8) Um gás ideal passou do estado A para o estado B, como mostra a figura ao lado. Sabendo
que 1 atm =105 N/m², calculemos o trabalho realizado pelo gás nessa transformação.
13
9) Um gás é comprimido por um agente externo, ao mesmo tempo que recebe um calor de 400 J de uma chama. Sabendo
que o trabalho do agente externo foi 700 J, vamos calcular a variação da energia interna do gás.
10) Sob pressão constante de 2,0 x105Pa, certa quantidade de gás ideal se expande, passando do volume V1 = 4,0 m3 para
V2 = 7,0 m3. Calcule o trabalho realizado pelo gás nessa transformação.
11) Determinada quantidade de gás ideal é comprimida por um agente externo ao mesmo tempo que recebe uma
quantidade de calor de 700 J. Sabendo que a variação da energia interna foi +900 J, calcule o trabalho realizado pelo gás.
12) Na figura abaixo vemos o gráfico p x V para certa quantidade de gás que sofre uma
transformação isotérmica à temperatura de 500 K. Qual é a pressão do gás no estado B?
13) Dentro de um cilindro munido de êmbolo móvel há determinada quantidade de gás
ideal. Inicialmente o gás ocupa volume VA = 200 litros e está à temperatura TA= 500 K e
sob pressão p = 2,0 x 105Pa. Mantendo-se a pressão constante, o gás recebe uma
quantidade de calor Q = 6,0 x 104 J, expandindo-se e passando a ocupar volume VB = 320 L.
a) Calcule o trabalho realizado pelo gás nessa transformação.
b) Calcule a variação da energia interna do gás nessa transformação.
14) Uma quantidade de calor Q = 400 J é fornecida a um gás ideal que está em um recipiente fechado, que não sofre
variação de volume. Calcule:
a) o trabalho realizado pelo gás;
b) a variação da energia interna do gás.
15) Um gás ideal executa o ciclo ABCDA indicado na figura. Para esse ciclo, calcule o
trabalho realizado pelo gás nas transformações:
a) AB
b) BC
c)CD
d) DA
e) ABCDA
b) a quantidade de calor trocada pelo gás na transformação ABCDA
c) a variação de energia interna sofrida pelo gás na transformação ABCDA
16) (E. Naval-RJ) A variação da energia interna, em joules, de um sistema constituído
por um gás ideal que, ao passar do estado inicial para o estado final, recebe um trabalho de 150 J e absorve uma
quantidade de calor de 320 J é igual a:
a) -170
b) 150
c) 170
d) 320
e) 470
Óptica
1) Num dia ensolarado, um menino e um poste projetam sombras de tamanhos 2,00 m
e 5,00 m, respectivamente.Calcule a altura do poste sabendo que a altura do menino é
1,60m.
2) Num determinado instante, o Sol, a Terra e a Lua estão nas posições indicadas na figura abaixo. São também
assinaladas as posições dos observadores A, B e C.
a) Qual dos observadores está na região da sombra?
b) Qual dos observadores está na região da penumbra?
c) Que tipo de eclipse ocorre para o observador A?
d) Que tipo de eclipse ocorre para o observador B?
e) Que tipo de eclipse ocorre para o observador C?
14
3) Um lápis de comprimento 16,0 cm é colocado em frente a uma câmara escura de orifício, como mostra a figura.
Calcule a altura h da imagem formada no fundo da câmara.
4) (Faap-SP) Um quadro, coberto com uma placa de vidro plano, não pode ser visto tão nitidamente quanto outro não
coberto, porque o vidro:
a) é opaco.
b) é transparente.
c) não reflete a luz.
d) reflete parte da luz.
e) é uma fonte luminosa.
5) Três esferas, X, Y e Z, quando iluminadas com luz branca, apresentam as cores branca, verde e azul, respectivamente.
Que cor essas esferas apresentam quando observadas através de um filtro que só permite a passagem da luz azul?
6) Uma camisa listrada tem listras brancas e azuis (puras). Em uma sala fechada, uma lâmpada emite luz vermelha pura.
Quais seriam as cores das listras nessa sala?
7) Um raio de luz incide em um espelho plano formando ângulo de 25°, como mostra a figura. Calcule:
a) o ângulo de incidência;
b) o ângulo de reflexão;
c) o ângulo a formado entre o raio refletido e o espelho;
d) o ângulo  entre os dois raios (refletido e incidente).
8) Um raio de luz passa por um ponto A, reflete-se em um espelho E e passa por um ponto B, como indica a figura
abaixo. Calcule:
a) a distância x;
b) o valor aproximado do ângulo formado entre o raio incidente e o espelho.
9) Na figura abaixo representamos uma fonte de luz F, um espelho E e cinco observadores situados nas posições A, B, C,
Oe G. Quais desses observadores podem ver a imagem de F?
15
10) Em uma das paredes de uma sala há um relógio de ponteiros no qual, em vez de números, há pequenos traços. Na
parede oposta àquela onde está o relógio existe um espelho plano. A figura abaixo mostra a imagem do relógio nesse
espelho. Observe a imagem e diga: que horas são?
11) Na figura abaixo representamos um espelho E, o olho O de um observador e os objetos A, B, C e D. Quais são os
pontos cujas imagens podem ser vistas pelo observador?
12) Em cada figura a seguir temos um objeto AB diante de um espelho esférico de foco F. Determine as imagens.
13)(UF-RN)Um objeto real está a 60 cm de um espelho esférico côncavo. A imagem desse objeto é real, invertida e se
localiza na mesma posição do objeto. A distância focal do espelho, em cm, é:
a) 10
b)12
c) 15
d)30
e) 60
14) (UF-CE)Uma árvore de natal está enfeitada com algumas bolas de superfície externa refletora.Uma criança aproxima
e afasta de uma das bolas um pirulito disposto verticalmente. A respeito da imagem formada, podemos afirmar:
a) pode ser real ou virtual,dependendo da posição do pirulito.
b) é virtual,direita e reduzida, qualquer que seja a posição do pirulito.
c) é real, invertida e aumentada, qualquer que seja a posição do pirulito.
d) é virtual, invertida e aumentada, qualquer que seja a posição do pirulito.
e) é real, direita e reduzida, qualquer que seja a posição do pirulito.
16
15) (Vunesp-SP) Um pequeno prego se encontra diante de um espelho côncavo, perpendicularmente ao eixo óptico
principal, entre o foco e o espelho. A imagem do prego será:
a) real, invertida e menor que o objeto.
b) virtual,invertida e menor que o objeto.
c) real, direita e menor que o objeto.
d) virtual,direita e maior que o objeto.
e) real, invertida e maior que o objeto.
16) Em cada caso a seguir são dadas as características da imagem de um objeto real colocado em frente a um espelho
esférico. Em cada caso, diga se o espelho é convexo ou côncavo.
a) Imagem real.
b) Imagem virtual e maior do que o objeto.
c) Imagem virtual e menor do que o objeto.
17) Um objeto de altura o = 6 cm está sobre o eixo principal de um espelho esférico convexo e a 36 cm do vértice.
Sabendo que o raio de curvatura do espelho mede 24 cm, determine as características da imagem. São dados:o = 6 cm; p
= 36cm; R = 24 cm.
18) (Fuvest-SP) A imagem de um objeto forma-se a 40 cm de um espelho côncavo com distância focal de 30 cm. A
imagem formada situa-se sobre o eixo principal do espelho, é real, invertida e tem 3 cm de altura.
a) Determine a posição do objeto.
b) Determine a altura do objeto.
c) Construa o esquema referente à questão, representando objeto, imagem, espelho e raios utilizados e indicando as
distâncias envolvidas.
19) (FEI-SP) Deseja-se projetar uma imagem de um objeto real sobre uma tela, usando um espelho esférico que dista 6,0
m dela. A distância do objeto ao espelho deve ser igual a 25 em. A altura do objeto é de 2,0 em. Determine:
a) o tipo do espelho e sua distância focal;
b) o tamanho da imagem e se ela será direita ou invertida.
20) (F.M. Vassouras-RJ) Uma pessoa, estando a 20 em de um espelho esférico côncavo, observa a imagem do seu
próprio rosto refletido no espelho. A imagem é uma vez e meia maior do que o rosto da pessoa. Nesta situação, quanto
vale, aproximadamente, o raio de curvatura do espelho?
a) 12 em
b) 30 em
c) 40 em
d) 60 em
e) 120 em
21) Um objeto real, de altura 12 em, está a 18 em do vértice de um espelho convexo cujo raio de curvatura mede 18 em.
Determine:
a) a distância focal do espelho;
b) a posição da imagem;
c) o tamanho da imagem.
22) Um objeto real é colocado diante de um espelho esférico que fornece uma imagem virtual desse objeto. A altura do
objeto é o triplo da altura da imagem e esta encontra-se a 10 em do vértice do espelho.
a) A imagem é direita ou invertida?
b) Qual a abscissa do objeto?
c) Qual a distância focal do espelho?
d) O espelho é côncavo ou convexo?
23) Sobre um anteparo, a 48 em de distância de um espelho esférico, é projetada a imagem de um objeto, ampliada duas
vezes. Determine:
a) a posição do objeto;
b) a distância focal do espelho e sua natureza.
17
24) Na tabela abaixo temos as velocidades da luz em um tipo de vidro, para uma luz verde monocromática e uma luz
vermelha monocromática.Calcule:
Luz
verde
vermelha
Velocidade 1,970.108m/s 1,985.108 m/s
a) o índice de refração do vidro para a luz verde;
b) o índice de refração do vidro para a luz vermelha.
25) O índice de refração da água para a luz amarela do sódio é n = 1,33. Calcule a velocidade dessa luz na água.
26) Um raio de luz monocromática passa de um meio A para um meio 8, como mostra a figura. Os índices de refração
dos meios A e B para essa luz são nA = 3 .e nB = 2,5, respectivamente. Calcule:
a) o ângulo (X formado entre a normal e o raio refratado;
b) a velocidade da luz em cada um dos dois meios.
27) (Vunesp-SP)A figura mostra a trajetória de um raio de luz que se dirige do ar para uma substância X. Sabendo que
sen 48° = 0,74 e sen 30° = 0,50, o índice de refração da substância X em relação ao ar é:
a) 0,67
b) 0,90
c) 1,17
d) 1,34
e) 1,48
28) Na figura abaixo representamos uma bolinha B que está no fundo de um recipiente contendo água. Um observador,
cujo olho está na posição O, vê a bolinha na posição B'. Trace o raio de luz que parte de B e atinge o olho do observador.
29) Um raio de luz monocromática, propagando-se inicialmente no ar, incide na
água com ângulo de 50°, formando-se um raio refletido (x) e um raio refratado (y).
Sabe-se que o índice de refração da água (para essa luz) é igual a 1,33.
Consultando uma tabela trigonométrica ou utilizando uma calculadora eletrônica,
determine os valores aproximados de:
a) ângulo formado entre o raio refratado e a normal;
b) desvio sofrido pelo raio incidente (i) ao passar para a água;
c) ângulo entre o raio refletido e o raio refratado.
18
30) Em cada figura a seguir temos um objeto real o, colocado em frente a uma lente esférica delgada. Determine as
imagens.
31) Em cada caso a seguir são dadas as características da imagem de um objeto real colocado em frente a uma lente
esférica delgada. Em cada um deles, diga se a lente é convergente ou divergente.
a) Imagem virtual e menor do que o objeto.
b) Imagem virtual e maior do que o objeto.
c) Imagem direita e menor do que o objeto.
d) Imagem direita e maior do que o objeto.
e) Imagem real.
f) Imagem invertida.
32) (UF-PI) A figura representa um objeto y diante de uma lente delgada
convergente cujos focos são F e F'. Pode-se afirmar que a imagem desse objeto
será:
a) virtual, invertida, maior que y.
b) virtual, invertida, menor que y.
c) real, direita, menor que y.
d) real, invertida, igual a y.
e) real, direita, maior que y.
33) Um objeto real de altura 12 cm está colocado a 20 cm de uma lente divergente
cuja distância focal tem módulo 10 cm.
a) Qual a distância da imagem à lente?
b) A imagem é real ou virtual?
c) A imagem é direita ou invertida?
d) Calcule o tamanho da imagem.
34) (Vunesp-SP) Sobre o eixo de uma lente convergente, de distância focal 6,0 cm, encontra-se um objeto, afastado 30
cm da lente. Nessas condições, a distância da imagem à lente será:
a) 3,5 cm
b) 4,5 cm
c) 5,5 cm
d) 6,5 cm
e) 7,5 cm
35) (Cesupa-PA) Um objeto real de 6 cm de altura é colocado perpendicularmente ao eixo principal de uma lente
divergente de distância focal igual a 150 cm. Se a imagem conjugada é direita e três vezes menor que o objeto, então a
distância do objeto à lente, em centímetros, é:
a) 100
b) 200
c) 300
d) 400
e) 500
Vestibular
1) (Fuvest-SP) Através do espelho (plano) retrovisor, um motorista vê um caminhão que viaja atrás do
seu carro. Observando certa inscrição pintada no pára-choque do caminhão, o motorista vê a seguinte imagem:
Pode-se concluir que a inscrição pintada naquele pára-choque é:
2) (UF-ES) Quando aproximamos um objeto de um espelho côncavo:
a) sua imagem real diminui e afasta-se do espelho.
b) sua imagem real diminui e aproxima-se do espelho.
c) sua imagem real aumenta e afasta-se do espelho.
d) sua imagem real aumenta e aproxima-se do espelho.
e) sua imagem real não se altera.
3) (Cesgranrio-RJ) Um objeto colocado muito além de C, centro de curvatura de um espelho esférico côncavo, é
aproximado vagarosamente do mesmo. Estando o objeto colocado perpendicularmente ao eixo principal, a imagem do
objeto conjugada por este espelho, antes de o objeto atingir o foco, é:
a) real, invertida e se aproxima do espelho.
b) virtual, direita e se afasta do espelho.
19
c) real, invertida e se afasta do espelho.
e) real, invertida, fixa num ponto qualquer.
d) virtual, invertida e se afasta do espelho.
4) (U. E. Londrina-PR) O esquema abaixo representa um espelho esférico côncavo de
pequena abertura, seu eixo principal e os raios incidentes r1 e r2. Pelas indicações do
esquema, a imagem de um objeto real apoiado sobre o eixo principal, formada pelo
espelho, será virtual:
a) somente se o objeto estiver entre M e N.
b) somente se o objeto estiver entre N e P.
c) somente se o objeto estiver além de P.
d) se o objeto estiver no ponto P.
e) qualquer que seja a posição do objeto.
5) (U. E. Londrina-PR) Um instrumento óptico conjuga, a um objeto real, uma imagem maior que ele. Este instrumento
pode ser:
a) uma lente divergente.
b) um espelho plano. c) um espelho convexo.
d) uma lente convergente.
HIDROSTÁTICA
P
F
A
d
m
V
f
F

A a
P = d.g.h
d1..h1= d2 . h2
E = dL.g.Vd
TERMOLOGIA
ºF = 1,8ºC + 32
L = Lo .  . t
K= ºC + 273
A = Ao . . t (onde =2)
Q = m . c. t
C
-[Qfor] = [ Q rec]
Q
t
V = Vo .  . t ( onde =3
C=m.c
L
Q
m
 -[ m . c. (Tf – Ti)] = m . c. (Tf – Ti)
GASES
P1 P 2

T1 T 2
V1 V 2

T1 T 2
P1 . V1 = P2 . V2
P1.V 1 P 2.V 2

T1
T2
P . V = n . R .T
TERMODINÂMICA
 = P ( Vf – Vi)
Q = + U
ÓPTICA
n
c
V
V 1 sin 1 n 2


V 2 sin
n1
n1 . sin1 = n2 . sin2
1
1
1


f
Di Do
A
i
 Di

O
Do
20
QUÍMICA
Mol
As questões de números 1 o 3 referem-se à substância cloro (Cl2),de larga utilização industrial no tratamento de águas e na obtenção
de matérias-primas clorados.
1) O elemento químico cloro é formado por dois isótopos: cloro-35e cloro-37. Determine a mossa atômico de cada isótopo.
2) A ocorrência natural dos isótopos do cloro é 75% de cloro-35 e 25% de cloro-37. Indique os cálculos que permitem obter a massa
atômica do elemento.
3) Nos cálculos químicos,o valor da massa atômica do elemento cloro é igualo 35,5 u. Em função disso, qual é o massa molecular da
substância cloro?
4) A glicose, açúcar produzido na fotossíntese, possui fórmula C 6H1206. Quantas moléculas H20, reunidas, teriam a mesma massa que
uma molécula de glicose?
5) Determine a massa, em unidades u, dos íons NH4+, N03-, So2-4 e P03-4
6) Admita que um lápis tenha a massa de 5 g. Determine a massa de um mol de lápis e compare o resultado com a massa da Terra(6,0
x 1021 toneladas).
7) Determine o número de átomos de mercúrio em um termômetro que contenha 8,0 g desse metal.
8) Em alguns termômetros, o mercúrio é substituído por um líquido avermelhado, uma mistura de álcool com iodo, denominada
álcool iodado. Determine a massa do álcool iodado que contém 0,5 mol de etanol (C 2H60) e 0,02 mol de iodo(I2),
9) Determine o número de átomos em 4 mols de moléculas de ozônio, O3
10) Qual é a massa de ferro em um prego que contém 6,0 x 10²² átomos desse metal?
11) Determine a quantidade de mols existente em um litro de água líquida. (Dado: densidade da água líquida= 1,0g/mL) ,
12) Quantas moléculas existem em 132g de gelo-seco (C02sólido)?
13) Sabendo que em 100g de ouro18 quilates há 75 g de ouro, quantos átomos desse metal haverá em uma jóia feita com 2 g de
ouro18 quilates?(Massa atômica do ouro:200 u)
14) A barrilha (Na2C03)é utilizada no tratamento da água que abastece as comunidades. Sabendo que uma estação de tratamento
utilizou 2,12 toneladas desse sal, determine:
a) a quantidade de mols correspondente;
b) o número de íons Na+ presente naquela massa.
15) O sulfato de alumínio,Al2(S04)3 também é utilizado no tratamento da água. Determine o número total de íons em 68,4 kg desse
sal.
16) (UFAC)A massa molecular do composto Na2S04.3H20 é igual a:
(Dados:H= 1 u;O= 16u; Na= 23 u; S= 32 u)
17) (Vunesp-SP) Na Tabela Periódica atual, a massa atômica de cada elemento aparece como número não-inteiro porque:
a) há imprecisão nos métodos experimentais empregados.
b) é a média aritmética das massas atômicas dos elementos superior e inferior da mesma família.
c) é a média aritmética das massas atômicas dos elementos com igual número de prótons.
d) é a média ponderada das massas atômicas dos isótopos naturais do elemento.
e) é sempre múltipla da massa atômica do hidrogênio.
18) (Cesgranrio-RJ) Um frasco contém uma mistura de 16 g de oxigênio e 55 g de gás carbônico. O número total de moléculas dos
dois gases no frasco é de:
21
19) (UFAC) O número de mols existente em 160 g de hidróxido de sódio (NaOH) é:
(Dados:H = 1 g; O= 16g; Na = 23 g)
20) (PUCC-SP)O inseticida BHC tem fórmula C6H6Cl6. O número de átomos de cloro em 2,0 mol de moléculas do composto é igual
a:
Fórmulas
1) Determine a porcentagem em massa de cada elemento na glicose, C6H12O6,
2) A análise de um material revelou1,2x10 24átomos, de ferro e 3 mols de átomos de oxigênio. A porcentagem em massa de ferro
nesse composto será igual a:
a) 90%
b) 85%
e) 70%
d) 75%
c) 80%
3) Sólido branco, viscoso, suficientemente flexível para ser trabalhado na produção de mancais e no recobrimento de fios elétricos e
panelas, o teflon é um polímero de tetrafluoro-etileno, representado pela molécula orgânicaF2C = CF2. Qual será a porcentagem em
massa do elemento químico flúor nessa molécula?
4) O propano, encontrado nos botijões a gás, apresenta 81,8% em massa de carbono, e a porcentagem restante é devida ao elemento
hidrogênio. A fórmula mínima desse gás será:
5) No século XIX foi descoberto que tanto o benzeno, solvente de graxas, quanto o acetileno, usado atualmente em soldas,
apresentavam a mesma fórmula centesimal:
C92,3% e H7,7%
Conhecendo esse fato e após realizar alguns cálculos, um estudante afirmou:
I) Ambas as substâncias possuem a mesma fórmula mínima:CH
II)Ambas as substâncias devem possuir a mesma fórmula molecular.
III) O benzeno e o acetileno devem possuir propriedades químicas exatamente iguais.
IV) 1,0 kg de qualquer uma das substâncias deve conter 923 g de carbono.
Estão corretas somente as afirmações:
a) I e IV.
b) I e II. e) I e III.
d) II e IV.
c) III e IV.
6) A análise de um composto com 142 u de massa molecular revelou a seguinte composição: Fósforo 43,7% em massa, Oxigênio
56,3% em massa
Com base nessas informações, determine a fórmula molecular desse composto.
7) Um dos poluentes da atmosfera, emitido pelos motores dos carros, possui massa molecular igual a 46 u. Sabendo que a fórmula
porcentual desse poluente é N=30,4%, O=69,6%, determine sua fórmula molecular.
8) O ácido oxálico tem fórmula mínima igual a CHO 2 e massa molar igual a 90 g/mol. Qual é a fórmula molecular desse ácido?
9) Um composto de massa molecular igual a 266 u apresenta 34,59% de sódio, 23,30% de fósforo e 42,11 % de oxigênio. Qual é a
fórmula molecular desse composto?
10) Um composto cuja massa molecular é 400 u possui 28% em massa de ferro. Qual é o número de átomos de ferro na fórmula do
composto?
11) Um alceno, substância formada apenas por carbono e hidrogênio, possui 85,7% em massa de carbono. A análise dessa substância
revelou que em 35 g de sua massa há 3,0 x 10²³moléculas. Com base nessas informações, qual é a fórmula molecular do alceno?
Gases
1) No lugar do mercúrio, um cientista usou um líquido X no barômetro de Torricelli e observou que a coluna adquiria a altura de 50
cm ao nível do mar. Se a experiência fosse feita no alto de uma montanha, a coluna do líquido X seria maior ou menor? Porquê?
2) O que ocorrerá com o volume de um balão comum de borracha, cheio de ar, se você colocá-lo na geladeira?Porquê?
3) Um cilindro com êmbolo móvel contém 100 mL de CO 2 a 1,0 atm. Mantendo a temperatura constante, se quisermos que o volume
diminua para 25 mL, teremos de aplicar uma pressão igual a:
a) 5 atm. b) 4 atm.
c) 2 atm. d) 0,4 atm.
e) 0,1 atm.
22
4) Um gás está preso em um cilindro com êmbolo móvel. Mantendo-se a temperatura constante, se a pressão P do gás passar para 3P,
o volume V:
V
a) passará para
.
b) passará para 3V.
c) passará para V + 3.
d) passará para V - 3.
e) não sofrerá alteração.
3
5) Certa massa de gás neônio está encerrada em um cilindro com êmbolo móvel, ocupando um volume de 500 mL a 0,6 atm.
Mantendo-se a temperatura constante, se a pressão passar para 1,8 atm, o volume:
a) sofrerá redução de 30%.
b) passará para 1500 mL.
c) passará a ser 33% do valor inicial.
d) sofrerá expansão de 20%.
e) não sofrerá alteração.
6) Sem alterar a massa e a temperatura de um gás, deseja-se que um sistema que ocupa 800 mL a 0,2 atm passe a ter uma pressão de
0,8 atm. Para isso, o volume do gás deverá ser reduzido para:
a) 600 mL.
b) 400 mL.
c) 300 mL.
d) 200 mL.
e) 100mL.
7) A figura mostra dois balões interligados por uma torneira. A interligação tem volume
desprezível e no balão A a pressão vale 300 mmHg. Abrindo a torneira e mantendo a temperatura
constante, qual será a pressão final do sistema?
8) Um cilindro com êmbolo móvel contém 600 cm³ de gás hélio a 27 ºC. Aquecendo o cilindro até
127 °C, sob pressão constante, qual será o novo volume ocupado pelo gás?
9) Para que temperatura Celsius deveremos esfriar 10,0 L de gás neônio,inicialmente a 27 °C, de modo que o volume seja reduzido
para3,0 L, sob pressão constante?
10) No rótulo de um frasco há a seguinte inscrição: 'Pressão interna igual a 2,4 atm (27 °C). Não aqueça acima de 127 °C'. Qual a
pressão máxima provável que o frasco pode suportar?
11) Quantos mols de gás hélio irão ocupar um cilindro de êmbolo móvel de 1,0 L, sob 0,82 atm e 27 °C de temperatura?
12) Em um air bag,a colisão do carro aciona um mecanismo que faz ocorrer a reação:
2NaN32Na(s)+ 3N2(gás)
Qual o volume ocupado por um airbag com 168 g de N2,sob pressão de 1246,4 mmHg e 27 °C de temperatura?(Dado:N = 14u)
13) Um balão foi preenchido com ar a 127 °C e 0,82 atm, ocupando um volume de 20 m³. Supondo que o ar funcione como um gás
de massa molar igual a 29 g/mol, qual a massa de ar dentro do balão?
14) Considerando as condições normais de temperatura e pressão(CNTP), calcule a massa do gás hélio correspondente a 5,6 L.
(Dado:Massa molar do hélio= 4 g/mol)
15) Determine o volume molar de um gás ideal em condição ambiente (25 °C, 1 atm).
16) Foi solto um balão com 20 m³ de ar aquecido, sob pressão de 1,6 atm e temperatura igual a 127 ºC. Em certa altitude, a pressão
interna do ar caiu para 1,4 atm e o volume foi reduzido para 16 m³. Com base nessas informações, qual a temperatura, em graus
Celsius, do gás do balão naquela altitude?
17) Em um dia muito frio, uma pessoa inalou 450 mL de ar a 760 mmHg e -3 ºC. Qual o volume ocupado por essa massa de ar nos
pulmões a 37 °C e 762 mmHg?
18) (UnB-DF)Um balão que contém gás oxigênio (02), mantido sob pressão constante, tem volume igual a l0 L, a 27°C. Se o volume
for dobrado, podemos afirmar que:
a) a temperatura em °C dobra.
b) a temperatura em K dobra.
c) a temperatura em K diminui à metade.
d) a temperatura em °C diminui à metade. e) a temperatura em K aumenta de 273 K.
19) (FCMSC-SP) Em um dia de inverno, à temperatura de 0 °C, colocou-se uma amostra de ar, à pressão de 1,0 atm, em um
recipiente de volume constante. Transpondo essa amostra para um ambiente de 60ºC, que pressão ela apresentou?
20) (Unifor-CE) Um recipiente de 24,6 L contém 1,0 mol de nitrogênio exercendo a pressão de 1,5 atm. Nessas condições, a
temperatura do gás vale, na escala Kelvin:
23
Estequiometria
1) A queima completa de fuligem (carbono) produz gás carbônico. Calcule a massa de CO 2 obtida na queima de 50 mols de fuligem,
de acordo com a equação: C + O2  CO2.
2) A equação química N2+ 3H2  2NH3 caracteriza o método Haber, processo industrial de produção de amônia, matéria-prima da
indústria de fertilizantes. Qual é a quantidade de mols de H 2 necessária para produzir 1,7 kg de amônia?
3) A combustão completa do gás de isqueiro, butano (C4H10}ocorre de acordo com a equação:
13
C4Hl0+
02 4C02+ 5H20
2
Na combustão de 0,58 g de gás de isqueiro, determine:
a) a massa de gás O2consumido;
b) o volume de gás CO2 formado nas CNTP.
4) A produção de cal virgem é realizada pela decomposição térmica (pirólise) do calcário: CaC03  CaO+ CO2
Qual massa de cal virgem poderá ser obtida a partir de 1,0 tonelada de CaC03?
5) Admitindo que o Brasil produza 4,9x10 9 kg de ácido sulfúrico pelo processo: 2S + 2H20 + 302  2H2S04
Calcule a massa consumida de enxofre.
6) A água oxigenada, solução aquosa de H202, é guardada em frascos escuros para evitar a decomposição:H 202H20 + 1/202
Com base nessas informações, calcule o volume de O2 que seria produzido pela decomposição de 170g deH202 nasCNTP.
7) Uma das reações de destruição do ozônio pelo cloro dos CFCs é dada por:
03(g) + Cl(g) 02(g) + ClO(g)
Calcule o número de moléculas de ozônio destruídas por 71 g de átomos de cloro.
8) Um caminhão derramou acidentalmente na estrada uma solução contendo 730 kg de ácido clorídrico. Para neutralizar o ácido, o
Corpo de Bombeiros jogou CaC03, que reage de acordo com o seguinte processo:
CaC03+ 2HCI CaCl2 + CO2 + H20
a) Calcule a massa de CaC03 que neutralizará o ácido clorídrico.
b) Determine o volume de gás carbônico produzido a 25 °Ce 1 atm.
(Dado:volume de 1 mol deCO2 a 25 °C e 1 atm== 24 L)
9) A produção de alumínio, através de Al203 presente na bauxita, pode ser equacionada por:
Al203+ 3C2Al+ 3CO
Com base nessas informações, responda:
a) Qual massa de alumínio será obtida a partir de 510t deAl2O3?
b) Que volume de CO, um gás muito tóxico, será produzido a partir de 1,0 mol de A1 203, nas CNTP?
10) Considere o oxigênio contido em 1000 L de ar a 25 °C e 1 atm. Determine a massa de metano (CH4), importante gás combustível
fóssil, que poderá ser queimada por esse gás oxigênio, de acordo com a equação:
CH4+ 202 CO2+ 2H20
(Considere: volumedeO2= 20% do volume do ar; volume molar do O2 a 25 °C e 1 atm = 25 L/mol)
11) O processo descrito a seguir é um dos que ocorrem na formação de chuvas ácidas:
2S02(g) + 02(g) 2S03(g) , Após a reação completa entre 200 mols de S02 e 150 mols de O2, haverá:
a) formação de 200 mols de S03 e excesso de 50 mols de S02
b) formação de 100 mols de S03, sem excesso de reagente.
c) formação de 350 mols de S03, sem excesso de reagente.
d) formação de 200 mols de S03 e excesso de 50 mols de O2,
e) formação de 150 mols de S03 e excesso de 50 mols de O2
12) Nos conversores catalíticos dos carros, um dos processos que ocorrem é: CO+ ½ 0 2 CO2
Se misturarmos 56 g de CO com 50 g de O2, ao término da reação haverá excesso de:
a) 18 g de O2
b) 18 g de CO
c) 16 g de CO
d) 32 g de O2
e) 10 g de O2
24
13) Na produção de amônia ocorre a reação: N2(g)+ 3H2(g)  2NH3(g)
Partindo de 500 L de gás N2, quais serão os volumes dos demais gases participantes da equação química mantidos sob pressão e
temperatura constantes?
14) No conversor catalítico dos carros, cada litro de CO necessita de 0,5 L de gás oxigênio para converter se em 1,0 L de gás
carbônico. Nas mesmas condições de pressão e temperatura, qual volume de CO 2 será obtido após a reação completa da mistura com
10L de CO e 10L de O2?
15) Num acidente, 4,9 toneladas de ácido sulfúrico são derramadas numa rodovia.Quantas toneladas de óxido de cálcio devem ser
utilizadas para neutralizar o ácido? (Dados:equação da reação: H2S04 + CaOCaS04 + H20;
massa molar do H2S04= 98 g/mol;
massa molar do CaO = 56 g/mol)
16) (UEl-PR) A questão a seguir refere-se à obtenção de 56,0 toneladas de ferro metálico pela reação representada pela equação:
Fe203(s)+ 3C(s)  3CO(g) + 2Fe(s)
(Dados:massa molardo Fe= 56,0 g/mol; massa molar do Fe203= 160 g/mol)
Que quantidade de monóxido de carbono, em mols,é obtida?
17) (Cesgranrio-RJ) Um funileiro usa um maçarico de acetileno para soldar uma panela.O gás acetileno é obtido na hora, através da
seguinte reação química:
CaC2+ 2H20 Ca(OH)2+ C2H2
Qual a massa aproximada de carbureto de cálcio (CaC2) que será necessária para obter 12,3 L de acetileno(C2H2)a 1 atm e 27°C?
(Dados:Ca = 40; C = 12)
Soluções
1) Em um balão volumétrico de 400 mL, são colocados 18 g de cloreto de sódio e água suficiente para atingir a marca do gargalo. A
concentração dessa solução será igual a:
a) 18 g/mL.
b) 18g/L.
c) 45 g/mL.
d) 45 g/L.
e) 4,5 g/L.
2) A secreção média de lágrimas de um ser humano é de 1 mL por dia. Admitindo que as lágrimas possuam sais com concentração de
6 g/L, indique a massa de sais perdida em um dia.
a) 0,6 mg
b) 0,6 g
c) 0,06 g
d) 0,06 mg
e) 0,006 g
3) A concentração de sais na água do mar, em média, é igual a 35 g/L. Em uma salina, um tanque com dimensões de 10m x 5 m x 1
m foi completamente preenchido com água do mar. Após a evaporação, a massa de sal que restou no tanque foi de:(Dado:1 m³=
1000 L)
a) 1750 g
b) 17,5 kg
c) 350 kg
d) 700 kg
e) 1750 kg
4) O suco de laranja contém açúcares com concentração em torno de 104 g/L. Admitindo que o suco de duas laranjas seja suficiente
para encher um copo com 200 cm³, determine a massa média de açúcares em cada laranja.(Dado: 1 L = 1000 cm³)
5) Um químico dissolveu 60 g de açúcar em água suficiente para 800 mL de solução.Em seguida colheu uma amostra de10mL dessa
mistura. Qual é a massa de açúcar contida na amostra?
6) Admitindo que 240 mL de suco de laranja contenham 480 mg de íons potássio, determine a concentração do potássio neste suco
em gramas por litro.
Porcentagem em massa, ppm e ppb
7) Uma xícara contém 90 g de café com leite.Considerando que você adoce essa mistura com duas colheres de chá contendo 5 g de
açúcar cada uma, a porcentagem em massa de açúcar comum será:
a) 12,5%.
b) 6,25%.
c) 25%.
d) 10%.
e) 5%.
8)Uma salmoura, mistura de água e sal, contém 10% de NaCl.Em 1,5 kg dessa mistura, você encontrará:
a) 150 g de água.
b) 1350 g de sal.
c) 1350 g de água.
d) 0,15 g de sal.
e) 1500 g de água.
25
Concentração em mols/L
9) Um químico pesou 20,2 g de KN0 3, colocou essa massa em um balão volumétrico e adicionou água suficiente para obter 400 mL
de solução. Com base nessas informações, indique a concentração molar da solução acima.
a) 0,1 mol/L
b) 0,2 mol/L
c) 0,3 mol/L
d) 0,4 mol/L
e) 0,5 mol/L
10) A bateria de um carro possui uma solução aquosa de ácido sulfúrico, cujo concentração é representada por [H 2S04] = 5,0 M. Isso
significa que:
a) cada litro de solução contém 5,0 mol de H2S04
b) cada litro de solução contém5 g deH2S04
c) cada mililitro de solução contém 0,5 g de H2S04
d) cada mililitro de solução contém 5,0 mol de H2S04
e) cada litro de solução contém 9,8 g de H2S04
11) O rótulo de um frasco de laboratório traz a seguinte anotação: [HCl] = 1,0 M. Com base nessa informação, determine o volume a
ser recolhido dessa solução, tal que haja 3,65 g de HCl.
12) De modo geral, o nível máximo de íons cloreto na água potável corresponde a 250 mg/L. Esse valor equivale a uma concentração
molar igual a:
13) Uma solução de 0,8 M de CaCl2 apresenta:
a) [Ca2+] = 1,6 M e [Cl-] = 0,8 M.
b) [Ca2+] = [Cl-] = 0,8 M.
2+
d) [Ca ] = 0,8 M e [Cl ] = 1,6 M.
e) [Ca2+] = [Cl-] = 0,4 M.
c) [Ca2+] = [Cl-] = 1,6 M.
14) Uma solução aquosa de K3P04apresenta 0,6 mol/L, em relação aos cátions K+. A concentração molar dessa solução em relação ao
sal será de:
a) 0,1 mol/L.
b) 0,2 mol/L.
c) 0,3 mol/L.
d) 0,4 mol/L.
e) 0,5 mol/L.
15) Sabendo que a água do mar apresenta, em média, uma concentração molar de íons Na + igual a 0,46 mol/L, determine a massa de
íons Na+ em um copo comum (200 cm3) de água do mar.
16) Em condição ambiente, o gás oxigênio dissolve-se em água até a concentração de 0,05 g/L. Determine essa concentração em
mol/L. (Dado: massa molar do gás oxigênio = 32 g/mol)
17) As baterias novas possuem solução aquosa de H 2S04 a 38% em massa e densidade de 1,3g/mL. A concentração molar dessa
solução vale:
a) 1,5 M.
b) 5,0 M.
c) 7,0 M.
d) 0,005 M.
e) 2,5 M.
18) As soluções aquosas concentradas de sal de cozinha são chamadas de salmouras. Considere que uma salmoura tenha densidade
igual a 1,2 g/cm³ e possua20% em massa de sal. A concentração da salmoura será de:
a) 60 g/L.
b) 120 g/L.
c) 240 g/L.
d) 300 g/L.
e) 360 g/L.
19) Uma solução de H2S04 cuja concentração é igual a 4,9 g/L terá concentração molar igual a: (Dado:
massamolardeH2S04= 98 g/mol)
a) 0,05 mal/L.
b) 0,1 mal/L.
c) 0,2 mal/L.
d) 0,3 mal/L.
e) 0,4 mal/L.
20) A água de lavadeira.ou água sanitária pode ser considerada uma solução de hipoclorito de sódio (NaClO)a 5% em massa. Qual é
a concentração desse sal em g/L?(Dado:densidade da água de lavadeira.= 1,0 g/mL)
a) 10 g/L.
b) 20 g/L.
c) 30 g/L.
d) 40 g/L.
e) 50 g/L.
21) Sob a temperatura de 25°C, o ar é uma solução gasosa de densidade igual a 1,2 g/L. Determine a massa aproximada de ar em um
ginásio de esportes com dimensões de 40 m x 20 m x 10m. (Dado: 1m³= 1000 L)
22) Uma solução aquosa de cloreto de potássio, KCl, apresenta concentração molar igual a 1,0 mol/L.Determine o número total de
íons em 500mL dessa solução
23) A análise de uma amostra de água oxigenada revelou H 202 na concentração de 34 g/L. Determine a quantidade de mols de H 202
em um frasco com 50 mL dessa solução.
24) Nosso suco gástrico é uma solução aquosa de HCl, com concentração 36,5 .10 -2 g/L.Com base nessa informação, determine a
concentração molar do ácido clorídrico no suco gástrico.
26
Diluição
25) Considerando que 150 mL de solução de glicose, cuja concentração é igual a 270 g/L,foram diluídos para 450 mL de solução,
calcule o valor da concentração final:
26) Para diluir uma solução de HN03 tal que a concentração diminua de 2,0 M para 0,4 M, o que se deve fazer?
27) Sabendo que 8 cm³ de água destilada foram cuidadosamente adicionados a 2 cm³ de solução de H 2S04 3,0 M, determine a
concentração molar final da solução:
a) 0,8 M b) 0,4 M c) 0,6 M d) 3,5 M e) 3,8 M
28) Para diluir 100 mL de solução de HCl 1,8 M de modo que a concentração diminua para 0,3 M, você deverá adicionar:
a) 600 mL de água.
b) 500 mL de água.
c) 400 mL de água.
d) 300 mL de água.
e) 200 mL de água.
29) Um técnico de laboratório farmacêutico dispõe de 500 g de soro glicosado a 15%, mas deseja transformá-lo inteiramente em soro
a 5%. Para que isso ocorra, qual a massa de água destilada a ser adicionada?
30) Como as soluções ácidas são geralmente muito corrosivas, devem ser manuseadas com cautela. Um estudante colheu 5,0 cm³ de
solução 3,0 M de HCl e diluiu cuidadosamente com 95 cm³ de água destilada. A solução assim obtida tinha concentração igual a:
a) 0,0015 M.
b) 0,015 M.
c) 0,15 M.
d) 1,5 M.
e) 15 M.
31) Qual volume de água deve ser adicionado a 50 mL de solução de CaCl 2,de concentração 0,4 g/L, para que haja diminuição a 0,02
g/L?
a) 950 mL
b) 1 000 mL
c) 800 mL
d) 1 050 mL
e) 550 mL
32) Um soro glicosado foi diluído, tal que a porcentagem em massa de glicose diminuiu de 12% para 8%. Nessa condição, a relação
massa. Final de solução massa inicial de solução deve valer:
a) 3,0.
b) 2,5.
c) 2,0.
d) 1,5.
e) 1,0.
33) Como deve ser diluída uma solução para que a concentração comum diminua para 20% do valor inicial?
Misturas
1) A mistura de 200 mL de solução 1,5 M de NaOH com 300 mL de solução 0,4 M de H 2S04 resultará em solução final ácida, básica
ou neutra?
2) Considerando que 50 cm³ de solução de KOH foram titulados com 20 cm³ de solução 0,5 M de HCl, determine a concentração
molar do KOH.
3) 200 mL de solução 1,0 M de KOH foram misturados com 300 mL de solução 0,8 M de HCI.Com base nessas informações,
determine a concentração molar do ácido (ou da base) que sobrou após o término da reação.
4)Quais volumes de soluções de NaCl, de concentrações respectivamente iguais a 8 g/L e 4 g/L, devem ser misturados para que
sejam obtidos 200 cm³ de solução com concentração igual a 5 g/L?
5) Considere que 200 mL de uma solução de KCl a 100 ppm foram misturados com 300 mL de outra solução de KCl a 150 ppm.
Qual é a concentração da mistura final de KCI, em ppm? (Dado:1 ppm = 1,0mg/L)
6) Que volume de solução 0,1 M de NaOH deverá ser adicionado a 70 mL de solução 0,5 M do mesmo soluto para que seja obtida
uma solução de concentração igual a 0,38 M?
7) Que volume de solução 0,5 M de KOH neutralizará completamente 500 mL de solução 0,1 M de HN03?
8) A análise de um suco gástrico indicou que 2,5 mL de solução 0,02 M de NaOH neutralizam 5 mL da solução ácida (HCI). Qual é a
concentração molar de HCl naquele suco gástrico?
9) 15,0 g de um vinagre, solução aquosa de ácido acético (HAc), foram titulados com 50 mL de solução aquosa 0,20 M de
NaOH.Determine a porcentagem em massa de HAc na amostra. (Dado:
massa molar de HAc= 60 g/mol)
27
Vestibular
1) (Vunesp-SP) A massa de cloreto de cromio (III) hexaidratado necessária para se preparar1 L de uma solução que contém 20 mg de
Cr3+por mililitro é iguala:
a) 0,02 g.
b) 20 g.
c) 52 g.
d) 102,5 g.
e) 266,5 g.
2) (PUCC-SP)Evapora-se totalmente o solvente de 250 mL de uma solução aquosa de MgCl 2 de concentração igual a 8,0 g/L.
Quantos gramas de soluto são obtidos?
3) (UFRS) Há legislações que determinam que seja estabelecido nível de emergência quando a concentração de monóxido de
carbono atingir o valor de 4,6 . 10 4g de CO por m3 de ar. Ao ser estabelecido o nível de emergência, o número de moléculas de CO
presente em cada m³ de ar é de aproximadamente:
a) 104
b) 1012
c) 1017
d) 1021
e) 1023
4) (Fuvest-SP) O limite máximo de ingestão diária aceitável' (IDA) de ácido fosfórico, aditivo de alimentos, é de 5 mg/kg de massa
corporal.Calcule o volume de um refrigerante, contendo ácido fosfórico na concentração de 0,6 g/L, que uma pessoa de 60 kg deve
ingerir para atingir o limite IDA.
5) (FAAP-SP) Calcule a concentração em g/L, de uma solução aquosa de nitrato de sódio que contém 30 g de sal em 400 mL de
solução.
6) (UFRN) O volume de água, em mL, que deve ser adicionado a 80 mL de solução aquosa 0,1 M de uréia, para que a solução
resultante seja 0,08 M, deve ser igual a:
a) 0,8.
b) 1.
c) 20.
d) 80.
e) 100.
7) (Vunesp-SP) Pipetaram-se10 mL de uma solução aquosa de NaOH de concentração1,0 mol/L.Em seguida, adicionou-se água
suficiente para atingir o volume final de 500 mL. A concentração da solução resultante, em mol/L, é de:
a) 5,0 . 10-3.
c) 5,0 . 10-2.
e) 0,20.
-2
b) 2,0 . 10 .
d) 0,10.
8) (UFES)Submetendo-se 3 L de uma solução1M de cloreto de cálcio à evaporação, até um volume final de 400 mL, sua
concentração molar será igual a:
a) 3,00.
c) 5,70.
e) 7,50.
b) 4,25.
d) 7,00.
9) (Fuvest-SP) Se adicionarmos 80 mL de água a 20 mL de uma solução 0,20 M de hidróxido de potássio, iremos obter uma solução
de concentração molar igual a:
a) 0,010.
b) 0,020.
c) 0,025.
d) 0,040.
e) 0,050.
10) (OSEC-SP) Foram preparados 100mL de uma solução com 1 mol de KCl. Em seguido, foram tomados 50 mL dessa solução e
acrescidos 450 mL de água A molaridade da solução final será de:
a) 0,1.
b) 0,2.
c) 0,5. d) 1.
e) 10.
11) (UFPA)50 g de uma solução de H2S04 de 63% em massa são adicionados a 400 g de água. A porcentagem em massa de H2S04 na
solução obtida é de:
a) 7%.
b) 9%.
c) 10%.
d) 12%.
e) 16%.
12) (UFRJ) Um aluno deseja preparar 1500 mL de solução 1,4M de ácido clorídrico,diluindo uma solução 2,8 M do mesmo ácido.
a) Que volume da solução mais concentrada deve ser usado?
b) Que volume de água é necessário para esta diluição?
13) (FCC-BAA) 1 L de solução 0,10 mol/L de NaOH adiciona-se 1 L de solução 0,10 mol/L de HCI. Se
a solução resultante for levada à secura até que seja obtida uma massa sólida, esta deverá pesar:
a) 2,3 g.
b) 3,5 g.
c) 5,8 g.
d) 35 g.
e) 58 g.
14) (UFRJ)Um laboratorista dispõe de solução2 M de H 2S04 e precisa de uma solução 0,5 M desse ácido.
a) Determine que volume da solução inicial ele deve diluir para obter 200 mL da solução desejada.
b) Calcule a massa em gramas de H2S04 presente nos 200 mL da solução desejada.
c) Determine a concentração da solução inicial em gramas/litro.
28
15) (FESP)100 mL de solução aquosa de NaOH de concentração 0,1 mol/L neutralizam totalmente as
Seguintes soluções aquosas, exceto:
a) 25 mL de H2S04 0,2 mol/L.
b) 100 mL de H2S04 0,1 mol/L.
c) 50 mL de HCI0,2 mol/L.
d) 100mLde HCI0,1 mol/L.
e) 50 mL de HN030, 2 mol/L.
Termoquímica
1) Dado o diagrama de entalpia de um processo químico:
é possível afirmar que:
a) o processo é exotérmico com H= +230 kJ
b) o processo é endotérmico com H = +570 kJ
c) o processo é endotérmico com H = +230 kJ
d) o processo é exotérmico com H = -230 kJ
e) o processo é exotérmico com H = -570 kJ.
2) É característico de uma reação exotérmica apresentar:
a) reagentes e produtos líquidos.
b) entalpias iguais para reagentes e produtos.
c) apenas participantes gasosos.
d) entalpia dos produtos menor que a entalpia dos reagentes.
e) entalpia dos produtos maior que a entalpia dos reagentes.
3) O mercúrio pode ser obtido pela reação de cinóbrio, HgS (sulfeto de mercúrio), com oxigênio do ar. HgS (s) + O2(g)Hg(l)+ SO2(g)
H= - 238 kJ/mol
Com base nessas informações, responda:
a) A entalpia dos reagentes é maior ou menor que a entalpia dos produtos? Porquê?
b) Qual é o calor liberado na formação de 4,0 g de mercúrio?(Dado:Hg= 200)
4) A queima de 1 mol de carbono libera 94 kcal, enquanto a vaporização de 1 mol de água absorve 10 kcal. Qual massa de água
poderia vaporizar com a queima de 48g de carbono?(Dadas as massas molares:carbono = 12 g/mol; água= 18 g/mol)
5) Dada a equação termoquímica
Cgraf + 2S romb CS2(l) H= +19 kcal/mol
responda:
a) O processo é endo ou exotérmico?
b) Qual massa de enxofre rômbico seria necessária para produzir 1,2x1024 moléculas de sulfeto de carbono?(Dado:massa molar do
enxofre= 32 g/mol)
c) Com relação à questão anterior, qual é o valor de H envolvido no processo?
6) Observeo processo de vaporização da água:
H2O(l)  H2O(g) H = +44 kJ/mol
Determine o calor absorvido na vaporização de
0,9 g de água. (Dados:H = 1; O = 16)
7) Construa os diagramas de entalpia correspondentes às seguintes equações termoquímicas:
1
a) HgO(s)  Hg(l) + O2(g) H = +90 kJ/mol
2
1
b) Na(s) +
CI2-+ NaCI(s) H = -411 kJ/mol
2
A respeito do diagrama de entalpia:
Podemos afirmar que:
a) A formação de glicose é endotérmica.
b) O estado R tem entalpia1275 kJ.
c) O estado P tem entalpia zero.
d) 1mol de glicose tem entalpia igual a -1 275 kJ.
e) A entalpia da glicose é maior que a das substâncias simples formadoras .
29
8) A combustão do carbono grafite pode ser dada pela Seguinte equação:
Cgraf  O 2(g)  CO2(g) H= -394 kJ/mol
A queima de 3 g de grafite envolverá:
a) absorção de 197 kJ.
b) absorção de 98,5 kJ.
c) liberação de 1576 kJ.
d) liberação de 197 kJ.
e) liberação de 98,5 kJ.
9) Considere a equação termoquímica abaixo, a 25°C:
1
1
N2(g) + O2(g)  NO(g) H = +90kJ/mol
2
2
A reação de 4mol de N2(g)sob temperatura de 25°C, envolverá:
a) absorção de 180 kJ.
b) absorçãode360 kJ.
d) liberação de 180 kJ.
e) liberaçãode360 kJ.
c) absorçãode720 kJ.
10) Dadas as equações termoquímicas:
1
H2(g) + 02(g) H2O(g) H = -58 kcal
2
1
H2(g) + 02(g) H2O(l) H = -68 kcal
2
Determine a quantidade de calor envolvida na vaporização de 1 mol de água líquida.
11) Considerando os processos:
N2(g) + 3H2(g)  2NH3(g) H = -92 kcal
N2(g) + 202(g)  2N02(g) H = +68 kcal
1
H2(g) +
02(g)  H20(l) H = -286 kcal
2
Determine o H de combustão da amônia:
4NH3 + 702  4N02 + 6H20
12) Na respiração celular a glicose reage com o oxigênio de acordo com a equação:
C6H1206(g) + 602(g)  6C02(g) + 6H20(l)
Qual o H do processo?
Dados:
I) 6Cgraf + 6H2(g) + 302(g)  C6H1206(S) H = -1275 kJ
II) Cgraf + 02(g)  CO2(g) H = -394 kJ
1
III) H2(g) + 02(g)  H20(l) H = -286 kJ
2
13) Observe o diagrama ao lado:
Determine o H de combustão de CH4 dando CO.
14) A combustão incompleta de carbono, responsável pela produção do
monóxido de carbono, é difícil de ser realizada isoladamente em um
calorímetro. No entanto, o H desse processo pode ser calculado pelos seguintes dados:
Cgraf+ 02(g)  CO2(g) H = -394 kJ
2CO(g)+ 02(g)  2C02(g) H = -566 kJ
Com base nessas informações, determine o H da queima de 3 mols de de carbono no processo:
1
Cgraf +
02(g)  CO(g)
2
15) Calcule a entalpia do metano, em kJ/mol, a partir dos seguintes dados:
CH4(g) + 202(g)  CO2(g) + 2H20(l) H = -890 kJ/mol
Cgraf+ 02(g)  CO2(g) H = -394 kJ/mol
1
H2(g) + 02(g)  H20(l) H = -286 kJ/mol
2
30
Comentário: A entalpia do metano corresponde ao calor de formação:
Cgraf + 2H2(g)  CH4(g)
16) Em grandes centros urbanos, é possível encontrar uma coloração marrom no ar, decorrente da formação de gás N0 2 na reação
entre NO(g), produzido por motores a combustão, e oxigênio do ar:
2NO(g) + 02(g)  2N02(g)
Determine a variação de entalpia desse processo em kJ/mol de N02(g),
Dados:
1
1
N2(g) + 02(g)  NO(g) H = 90 kJ/mol
2
2
1
1
N2(g) + 02(g)  N02(g) H = 34 kJ/mol
2
2
17) No metano gasoso, a quebra de todas as ligações,produzindo átomos
isolados, apresenta um valor de H igual a:
a) +413 kJ.
b)-413kJ.
c) +4(413) kJ.
d) -4(413) kJ.
e) -2(413) kJ.
18) O gás acetileno possui a seguinte fórmula estrutural:
ligações da molécula desse gás será igual a:
a) 414 kJ.
b) 820 kJ.
c) 828 kJ.
d) 1234 kJ.
e) 1665 kJ.
A energia empregada para romper todas as
19) Utilizando energias de ligação, determine o H do processo envolvendo gases:
20) (UFV-MG) A “cal extinta” [Ca(OH)2]pode ser obtida pela reação entre óxido de cálcio (CaO) e água (H 20), com conseqüente
liberação de energia. O óxido de cálcio, ou “cal viva”, por sua vez, é obtido por forte aquecimento de carbonato de cálcio (CaC03).
As equações referentes a essas reações são:
I) CaO+ H20  Ca(OH)2+ calor
II)CaC03+ calor  CaO+ CO2
Identifique a afirmativa incorreta:
a) A reação II é endotérmica.
b) A reação II é uma reação de decomposição.
c) A reação I é uma reação endotérmica.
31
21) (Fuvest-SP) Considere os seguintes dados:
Reagente
Produto
H (condiçõespadrão)
I) C(gr)  C(d)
+0,5 kcal/moldeC
II) I(g)  2I2(g)
-25 kcal/moldeI
III)2C12(g) CI(g)
+30 kcal/mol de CI
Pode-se afirmar que o reagente tem maior energia do que o produto somente em:
a) I.
b) b) II c) III
d) I e II e) I e III.
22) (UFAL) Considere a seguinte equação termoquímica:
1
3
N2(g) + H2(g)  NH3(g) H=-46,0 kJ/mol
2
2
Pode-se, conseqüentemente afirmar que a formação de 2,0 mol deNH 3(g)consome:
a) 2,0 mol de H2, com liberação de calor.
b) 1,5 mol de H2, com absorção de calor.
c) 1,5 mol de H2, com liberação de calor.
d) 1,0 mol de N2, com absorção de calor.
e) 1,0 mol de N2, com liberação de calor.
23) (Fuvest-SP)A oxidação de açúcares no corpo Humano produz ao redor de4,0 kcal/g de açúcar oxidado. A oxidação de 0,1 mol de
glicose
(C6 e H12 06) vai produzir aproximada mente:
(Dados:H = 1,0; C= 12;O= 16)
a) 40 kcal.
b) 50 kcal.
c) 60 kcal.
d) 70 kcal.
e) 80 kcal.
24) (FEEQ-CEA) queima de 1,0 kg de metano (CH4) Liberou 5,5 . 104 kJ.Com base nesse dado,o calor Da combustão de1 mol de
metano é da ordem de :
(Dado:massa molar do CH4= 16 9 . mol-1)
a) 8,8 . 10-4
b) 8,8 . 10-3
c) 8,8 . 10-2
d) 8,8 . 102
e) 8,8 . 104
25) (FUC-MT) Analisando o diagrama abaixo
Podemos concluir a entalpia de formação
S03(g)é igual a:
a) +94,4 kcal/mol.
b) -94,4 kcal/mol.
c) -71,0 kcal/mol.
d)-23,4 kcal mol
e) +23,4 kcal mol
26) (Fuvest-SP) Considere a reação de fotossíntese a reação de combustão da glicose
representadas abaixo:
6CO2(g)
C6H12O6(S)
+
6O2(g)
+
6H2O(l)
C6H12O6(S) + 6O2(g)
6CO2(g) + 6H2O(l)
Sabendo que a energia envolvida na combustão de 1 mol de glicose é 2,8.106J, ao sintetizar 0,5 mol de glicose, a planta:
a) libera 1,4 . 106J.
b) libera 2,8 . 106J.
c) absorve 1,4 . 106J.
d) absorve 2,8 . 106J.
e) absorve 5,6 . 106J.
27)(Unicamp-SP) Uma vela é feita de um material ao qual se pode atribuir a fórmula C20H42. Qual o calor liberado na combustão de
10,0 g dessa vela à pressão constante? (Dadas as massas molares: C = 12 g/mol;
H = 1 g/mol)
61
C20H42(s) +
O2(g)  2OCO2(g) + 21 H2O(g)
2
H = -13 300kJ
28) (Fuvest-SP) Qual é a energia envolvida na obtenção de 10g de cobre metálico através da reação entre uma solução de sal de
cobre e zinco? O processo libera ou absorve energia?
(Dado: Cu= 63,5 g/mol)
Zn0 + Cu++  Cu0+ Zn++
H = -223 kJ
32
Cinética Química
1) Considere os seguintes processos:
I) Neutralização de leite de magnésia no estômago.
II)Oxidação do cobre, formando zinabre.
III) Ataque de ácido muriático (HCI) em pedaço de palha de aço. .
Apresenta(m) baixa energia de ativação somente o(s) processo(s): I
a) I.
b) II.
c) III. d) I e III.
e) II e III.
2) Observe o diagrama de energia a seguir: É correto afirmar que:
a) o processo é exotérmico.
b) a reação tem H = -30 kJ.
c) o complexo ativado é mais estável que os reagentes ou os produtos.
d) a energia de ativação vale -80 kJ.
e) a energia de ativação vale + 130 kJ.
3) Observeo diagrama abaixo:
Sobre ele um estudante afirmou:
I) O processo é endotérmico.
II) O H vale -226 kJ.
III) A energia de ativação vale +134 kJ.
Está(ão) correta(s) somente a(s) afirmativa(s):
a) I.
b) II.
c) III.
.
d) I e II. e) II e III
4) O holandês Jacobus Hendricus Van't Hoff (1852- 1911), um dos maiores
químicos de sua época, foi o primeiro a receber um prêmio Nobel de
Química, em 1901. Em um de seus trabalhos,Van't Hoff enunciou a seguinte
regra:um aumento de 10°C duplica a velocidade de uma reação
química.Verificou-se tempos depois que essa regra não era rigorosa. Mas,se
ela fosse verdadeira, cozinhar feijões em uma panela de pressão (110°C)
deveria:
I) consumir o dobro do tempo em relação ao cozimento em uma panela aberta a 100°C.
II) consumira metade do tempo em relação ao cozimento em uma panela aberta a 100°C.
III) consumir o mesmo tempo, qualquer que fosse o tipo de panela.
Está(ão) correta(s) somente a(s) afirmação(ões):
a) I.
b) II.
c) III.
d) I e III.
e) II e III.
5) O metal ferro reage comum a solução aquosa de HCl, originando gás hidrogênio e cloreto de ferro II. Assinale a alternativa que
indica a reação mais rápida entre o ferro e uma solução de HCl 1,0 M:
Fe(s)+ 2HCI(aq)  FeCI2(aq) + H2(g)
a) Um prego de ferro, a 25°C.
b) Um prego de ferro, a 40°C.
c) Ferro em pó, a 40°C.
d) Ferro, a 25°C.
e) Essa reação não depende da superfície de contato ou da temperatura.
6) Catalisador é:
a) uma substância que não possui participação na reação.
b) sinônimo de luz.
c) sinônimo de calor.
d) uma substância que acelera a reação e, no final, desaparece do sistema.
e) uma substância que acelera a reação e, no final, apresenta massa constante.
7) Assinale a alternativa que traz agentes que tendem a aumentar a velocidade de uma reação:
a) Calor, obscuridade, catalisador
b) Calor, maior superfície de contato entre reagentes, ausência de catalisador
c) Calor, maior superfície de contato entre reagentes, catalisador
d) Frio, obscuridade, ausência de catalisador
e) Catalisador e congelação dos reagentes
8) O diagrama corresponde ao processo: 2NH3(g) N2(g)+ 3H2(g) realizado com catalisador
e sem catalisador.
33
Com base nessas informações, é correto afirmar que:
a) O H da reação vale -92kJ
b) o processo catalisado apresenta H = + 92 kJ e energia de ativação + 255 kJ
c) O processo não catalisado tem energia de ativação +163 kJ
d) o processo não catalisado possui H = 171 kJ
e) O processo catalisado tem energia de ativação + 163 kJ
9) Dado o processo:
2HCl(g) +
<H2Cl2>
H2(g)
+
Cl2(g)
H = + 44 kcal
Complexo ativado
E sabendo que a energia de ativação vale 140kcal, construa um diagrama de energia com esses participantes, marcando corretamente
os valores de H e de energia de ativação.
10)Para a reação 203(g)  302(g), a energia de ativação é de aproximadamente 28 kcal. Com base nessa informação e sabendo que a
entalpia de formação do 03(g)vale +34 kcal/mol:
a) determine o valor do H desse processo;
b) construa um diagrama de entalpia mostrando a energia de ativação.
11) Considerando processo A B, construa um diagrama de energia.(Dados: H = -30 kJ;energia de ativação= +50 kJ)
12) Quais motivos você tem para mastigar bem os alimentos?
13) Explique a principal causa dos fenômenos abaixo:
a) Em minas de carvão, a presença de pó de carvão no ar pode provocar explosões.
b) Para evitar que a palha de aço enferruje, podemos esfregá-la em sabão.
c) Em uma churrasqueira,é mais fácil acender carvão em pequenos pedaços.
14) No processo de hidrogenação do acetileno produzindo etano, C2H2 + 2H2-C2H6, é possível afirmar que a velocidade:
a) de consumo de C2H2 é o dobro da velocidade de consumo de H2.
b) de consumo de H2 é igual à de formação de C2H6.
c) de consumo de C2H2 é diferente da velocidade de formação de C2H6.
d) de formação de C2H6 é a metade da velocidade de consumo de H2.
e) de consumo de C2H2 é igual à de consumo de H2.
15) É possível produzir benzeno a partir de acetileno por meio da seguinte reação, denominada trimerização: 3C2H2  C6H6
(Dados:VA= Vmédia do consumo do acetileno; VB= Vmédia de formação do benzeno)
Com base nessas informações, pode-se dizer que:
a) VA= VB
c) VA= 3VB
e) VB= 6VA
b) 3VA= VB
d) VA= 6VB
16) Observandoa tabela abaixo, os valores corretos de x e y devem ser:
N2
3H2  2NH3
Início
0,4 mol/L
0
Tempo =0
Formou
0,1 mol/L
Reagiu
X
Restou
Y
a) x = 0,05 mol/L; y = 0,35 mol/L.
b) x = 0,2 mol/L; y = 0,2 mol/L.
d) x = 0,4 mol/L; y = O.
e) x = 0,3 mol/L; y = 0,1 mol/L.
c) x = 0,1 mol/L; y = 0,3 mol/L.
17) Para determinada reação, verificou-se experimentalmente que a velocidade era proporcional ao quadrado da concentração de gás
hidrogênio:v = k[H2]². Assim,se a concentração de H2 for triplicada, a velocidade da reação deverá:
a) aumentar três vezes.
b) diminuir três vezes.
c) aumentar seis vezes.
d) aumentar nove vezes.
e) diminuir nove vezes.
18) O diagrama a seguir mostra as variações de concentração de duas substâncias, X e Y, participantes de uma mesma reação em
função do tempo, observe:
34
Com base no diagrama apresentado, responda:
a) Qual é o reagente? Por quê?
b) Qual é a velocidade média da reação, em função do reagente, no intervalo de tempo entre 0 e 10min?
c) Qual é a equação química corretamente balanceada?
Sugestão: Lembre-se de que os coeficientes serão proporcionais às quantidades de mols que reagem e se formam.
19) (Cesgranrio-RJ) O gráfico representa a variação das concentrações das substâncias X, Y e Z
durante a reação em que elas tomam parte.
Assinale a equação que representa a reação:
a)X+ ZY
b) X+YZ
c) X Y + Z
d)Y X+ Z
e) ZX+Y
20) (Fuvest-SP) O gráfico mostrado abaixo foi construído com dados obtidos no estudo da
decomposição de iodeto de hidrogênio, à temperatura constante.Em qual dos quatro trechos
assinalados na curva a reação ocorre com maior velocidade média? Explique.
21) (Cefet-RJ) Quando se leva uma esponja de aço à chama de um bico de gás, a
velocidade da reação de oxidação é tão grande que incendeia o material. O mesmo não
ocorre ao se levar uma lâmina de aço à chama. Nessas experiências, o fator que determina a
diferença das velocidades de reação é:
a) a pressão.
b) o catalisador.
c) o estado físico.
d) a concentração.
e) a superfície de contato.
22) (Vunesp-SP) Sobre catalisadores, quatro afirmações seguintes:
I) São substâncias que aumentam a velocidade de uma reação.
II)Reduzem a energia de ativação da reação.
III) As reações nas quais atuam não ocorreriam nas suas ausências.
IV) Enzimas são catalisadores biológicos.
Dentre estas afirmações, estão corretas, apenas:
a) I e II. b) II e III.
c) I, II e III.
d) I, II e IV.
e) II, III e IV.
23) (UFMG) Na cinética de uma reação, o aumento da temperatura provoca o aumento de todas as seguintes grandezas, exceto:
a) energia de ativação.
b) energia do sistema.
c) número de colisões entre as moléculas dos reagentes.
d) velocidade média das moléculas.
e) velocidade da reação.
24) (Fuvest-SP) O zinco reage com ácidos, ocorrendo liberação do gás hidrogênio.Adicionam-se quantidades iguais de ácido em duas
amostras de mesma massa de zinco, uma delas em raspas(A) e a outra em pó (B). Assinale o gráfico que deve representar a produção
de hidrogênio em função do tempo de reação para essa experiência.
25) (Cefet-MG) O diagrama abaixo representa a variação de energia ocorrida na combustão do álcool etílico, de acordo com a
equação:
C2H50H+ 302 2C02+ 3H20
35
Baseando-se no gráfico apresentado, é possível afirmar corretamente que:
a) a energia de ativação da combustão vale(a + b)kcal.
b) a energia liberada na combustão é (b) kcal/mol de álcool.
c) o complexo ativado tem conteúdo energético (a + b) kcal.
d) o H da reação de combustão é (b - a) kcal/mol.
e) o conteúdo energético dos reagentes é menor que o conteúdo energético dos produtos
da combustão.
26) (UFBA) Considere o diagrama abaixo para a seguinte reação:
Br + H2 HBr+ H
A entalpia da reação e a energia de ativação representadas são, respectivamente:
a) 3 kcal/mol e 28 kcal/mol.
b) 28 kcal/mol e 25 kcal/mol.
c) 28 kcal/mol e 3 kcal/mol.
d) 25 kcal/mol e 28 kcal/mol.
e) 25 kcal/mol e 3 kcal/mol.
27) (UFV-MG)A respeito de catalisador e catálise é incorreto afirmar que:
a) catalisadores são substâncias que participam das reações químicas, aumentando-lhes a velocidade, sem serem consumidas nas
reações.
b) a catálise é homogênea quando o sistema em reação e o catalisador formam um sistema homogêneo.
c) os catalisadores diminuem a energia de ativação da reação que catalisam.
d) a catálise é heterogênea quando o sistema em reação e o catalisador são necessariamente sólidos.
e) o H da reação não se altera porque o catalisador só participa dos estados intermediários da reação.
28) (UFMG) Analise o gráfico referente à reação: M+NP+Q, com temperatura e pressão constantes.
Entre as afirmativas que seguem, referentes ao gráfico, assinale a única errada.
a) X representa a energia de ativação da reação direta não catalisada.
b) Y representa a energia de ativação da reação direta catalisada.
c) Z representa o calor liberado na reação exotérmica.
d) A velocidade de reação representada pela curve I é menor que a representada
pela curva II.
e) O calor de reação correspondente à curva II é menor que o correspondente à
curva I.
29) (UFPA)Considerando o seguinte diagrama de energia para uma dada reação química:
pode-se afirmar que:
I) o H da reação é dado por Y.
II) a energia de ativação com catalisador é dada por X.
III) a reação é exotérmica.
IV) o abaixamento da energia de ativação, verificado pela adição de um
catalisador, é dado por W.
V) a reação é endotérmica.
Estão corretas somente as afirmativas:
a) I e V.
b) II e III e IV
c) II e IV.
d) III e IV.
e) II e V.
30) (Fuvest-SP)Dada a seguinte equação: reagentescomplexo ativado produto+calor. Represente em um gráfico(entalpia em
ordenada e caminho de reação em abscissa) os níveis das entalpias de reagentes, complexo ativado e produtos.
31) (UFMT)Estudar a rapidez com que as transformações químicas ocorrem é de extrema importância para os sistemas vivos,
geológicos e de produção.
36
Julgue os itens abaixo.
a) As substâncias que aceleram uma reação química, mas não são consumidas durante a reação, são chamadas de catalisadores.
b) Para a reação H2(g) + I2(g) 2HI(g) a expressão da velocidade é dada por v = k[H2] [I2].
c) Quanto menor a concentração dos reagentes, maior é a probabilidade de colisões efetivas (choques) entre suas partículas.
d) O aumento da temperatura acelera uma reação química, pois, em temperaturas mais altas, a energia cinética das partículas
reagentes é maior e, portanto, é maior a chance de elas se chocarem.
32) (UFMG) As curvas I e II representam caminhos possíveis para a reação de
hidrogenação do propeno.
a) Indique a curva que corresponde ao caminho da reação mais rápida.
b) Escreva o fator responsável por essa diferença de velocidade.
c) Compare as energias dos complexos ativados formados nos dois caminhos
da reação.
33) (Fuvest-SP) Na reação representada pela equação química N2+ 3H2 2NH3, o perfil energético, com e sem catalisador, é o
seguinte:
a) Calcule a energia envolvida por mol de NH3 formado. O processo é endotérmico ou exotérmico?
b) Calcule as energias de ativação para essa reação, na ausência e na presença de catalisador.
34) (UFMT) Nas madeireiras, o pó de madeira (serragem) pode ser queimado por uma faísca ou chama e produzir incêndios de
proporções incalculáveis. Quais os fatores que têm influência na velocidade da reação e justificam essa afirmação? Justifique sua
resposta.
Equilíbrio Químico
1) Considere as seguintes afirmações sobre equilíbrios químicos:
I) As velocidades das reações opostas são iguais.
II) No equilíbrio não existem reações químicas.
III) As concentrações dos participantes são iguais entre si.
IV)As concentrações dos participantes são constantes ao longo do tempo.
Está(ão) correta(s) somente a(s) afirmativa(s):
a) I e II.
b) I e III.
c) I e IV.
d) II e III
e) III e IV.
.
2) Determine a expressão de equilíbrio do processo: 30 2  203
3) No processo reversível N204  2N02 admita que a constante de equilíbrio seja igual a 4, em uma dada temperatura. Sabendo que a
concentração de N204 vale 0,5 mol/L, a concentração de N02 valerá:
a)
2 mol/L.
b) 2 mols/L.
c) 2,5 mols/L.
d) 3 mols/L.
e) 3,5 mols/L.
4) O equilíbrio 2A  B + D apresenta constante Kc= 0,25. Sabendo que, nesse equilíbrio, as concentrações molares de B e D são
iguais a 1,0 mol/L, a concentração da substância A deverá ser igual a:
a) 0,25 mol/L.
b) 0,5 mol/L.
c) 1,0 mol/L. d) 2,0 mols/L.
e) 4,0 mols/L.
37
5) Considere o processo 2S03(g)  2S02(g) + O2(g) . No equilíbrio,uma análise revela:
[S03] = 0,4 M; [S02] = 0,1 M; [02] = 0,2 M. Determine o valor da constante de equilíbrio (Kc) na temperatura do processo.
6) O equilíbrio H2(g) + Br2(g)  2HBr(g) a certa temperatura, pode ser representado
pelo diagrama da esquerda. Utilizando as informações do diagrama, determine o valor
da constante de equilíbrio.
7) O diagrama da direita
representa as condições
do equilíbrio PCl5(g) 
PCl3(g)+ C12(g) realizado
a certa temperatura.
Utilizando
as
informações
do
diagrama, determine o
valor da constante de
equilíbrio.
8) O diagrama abaixo mostra as variações de concentrações molares das substâncias A e B no equilíbrio AB.
a) Determine o valor de Kc.
b) Qual é o grau de decomposição da substância?
9) Noequilíbrio2NO+ Cl2 2NOCl,o aumento da concentração de NO provocará:
a) menor produção de NOCl
b) aumento da concentração de Cl2.
c) deslocamento do equilíbrio para a esquerda.
d) alteração da constante Kc.
e)formação de maior número de moléculas NOCl
10) Observe o diagrama abaixo, que descreve o equilíbrio 30 2  203,
O ponto A corresponde ao instante em que:
a) o equilíbrio foi rompido pela adição de 0 3,
b) o equilíbrio foi rompido pela adição de O2,I
c) é retirado O3 do sistema.
d) é retirado O2 do sistema.
e) é estabelecido um novo equilíbrio.
O ponto B corresponde ao momento em que:
a) um novo equilíbrio é estabelecido, após a adição de O 2,
b) um novo equilíbrio é estabelecido, após o acréscimo de 03,
c) o equilíbrio é rompido pela remoção de O2,
d) o equilíbrio é rompido pelo acréscimo de O 2,
e) a reação termina, após o consumo total de O2,
11) Dado o equilíbrio:
2NO(g)+ O2(g)  2N02(g) H= -114 kJ
Indique o deslocamento (para a direita ou esquerda) provocado pelas seguintes alterações:
a) Aumento da concentração de O2
b)Retirada de N02
c) Aumento da temperatura
d) Aumento da pressão
e) Aumento da concentração de 2N02
f)Retirada de NO
38
12) Qual é a ação do catalisador em um equilíbrio?Justifique sua resposta.
13) O equilíbrio 2CO(g) + 02(g)  2C02(g) H< 0 poderá ser deslocado para a direita efetuando-se:
a) o processo comum catalisador apropriado.
b) uma diminuição da pressão.
c) um aumento da concentração de CO2.
d) um aquecimento do sistema.
e) um aumento da concentração de O2,
14) Responda com base no diagrama a seguir,que se refere ao
equilíbrio, a 25°C: 2S03(g)  2S02(g)+ 02(g)
a) Determine o valor do abaixamento da energia de ativação para a
reação direta e para a reação inversa.
b) O que provocou esse abaixamento?
c) Qual é o H do processo?
d) Se a temperatura aumentar, qual será o sentido do deslocamento
do equilíbrio?
e) Se o volume for diminuído, para que lado o equilíbrio irá se
deslocar?
f) Escreva as expressões da Kc para esse equilíbrio.
15) Na água pura, a 25°C, tem-se [H+] = [OH-]= 1,0x10-7 mol/L. Com base nessas informações, podemos concluir que:
a) uma solução ácida terá [H+] < 1,0. 10-7 M.
b) uma solução básica terá [OH-] < 1,0' 10-7M.
c) não há íons OH- em uma solução ácida.
d) o 'limpa-forno', que contém praticamente apenas NaOH, apresenta [H +] > 1,0. 10-7M.
e) o suco gástrico, que possui grande quantidade de HCl, terá [H +] > 1,0' 10-7M.
16) Um alvejante de roupas, do tipo 'água de lavadeira', apresenta [OH -] aproximadamente igual a 1,0 . 10-4 mol/L. Nessas condições,
a concentração de H+ será da ordem de: (Dado: Kw= 10-14)
a) 10-2.
b) 10-3.
c) 10-10
d) 10-14.
e) zero.
17) Um xampu tem pH = 8. Podemos, então, concluir que:
a) o xampu é básico e possui [H+] = 1 . 10-8.
c) o xampu é ácido e possui [OH-] = 1 . 10-8.
e) o xampu é neutro e possui [H+] = 1 . 10-7.
b) o xampu é básico e possui [H+] = 1 . 10-6.
d) o xampu é ácido e possui [OH-] = 1 . 10-6.
18) Uma solução de pH = 2,5 terá concentração molar de íons H+ igual a:
a) 10-2.5. b) 10+2,5 c) 2,5 . 10-2.
d) 0,25. e) 2,5 . 10-2.
19) A análise de um suco de laranja revelou [H+] = 1,0 . 10-4 mol/L. Pode-se concluir sobre esse suco que:
a) é uma solução básica, com pH = 4.
b) é uma solução ácida, com pH= 10.
c) é uma solução ácida, com pH= 4.
d) é uma solução neutra.
e) terá [OH-] também igual a 1,0. 10-4 mol/L.
20) Veja, na tabela abaixo,o pH de alguns sistemas:
Com base nessa tabela, podemos concluir que:
a) I é o sistema mais básico.
b) a saliva não participa da digestão dos alimentos.
c) na água do mar tem-se [H+] > [OH-]
d) V é o sistema mais ácido, com pH < 7.
e) na urina tem-se [H+] > [OH-]
I
II
III
IV
V
Sistema
Urina
Saliva (em repouso)
Saliva (durante a refeição)
Água do mar
Ovos frescos
pH
6
6,4 - 6,9
7,0 - 7,5
7,8 - 8,3
7,6 - 80
21) A clara do ovo tem [OH-] = 1,0x10-6 mol/L. Sabendo disso, determine o valor do seu pH.
a) 8
b) 6
c) 4
d) 2
e) 1
22) A tabela abaixo fornece o intervalo aproximado de pH de alguns líquidos do nosso corpo.Observe:
Líquido Suco gástrico Saliva
Urina
Leite
Sangue
Lágrima
pH
1a3
6,4 a 6,9
4,8 a 7,5 (normal = 6)
6,6 a 7,6
7,3 a 7,5
7,3
39
Assinale a alternativa que traz os líquidos que estão na faixa das soluções alcalinas (básicas):
a) suco gástrico e sangue.
b) sangue e lágrima.
c) saliva e leite.
d) suco gástrico e saliva.
e) urina e saliva.
23)(UFRS)Uma reação química atinge o equilíbrio quando:
a) ocorre simultaneamente nos sentidos direto e inverso.
b) as velocidades das reações direta e inversa são iguais.
c) os reagentes são totalmente consumidos.
d) a temperatura do sistema é igual à do ambiente.
e) a razão entre a concentração de reagentes e produtos é unitária.
24) (UFMG) Considere a reação hipotética: A + B  C + D,
Considere,também, o gráfico
Com base nessas informações, assinale a única afirmativa incorreta:
a) No instante inicial, a velocidade V1 é máxima.
b) No instante inicial, as concentrações de C e D são mínimas.
c) No instante x, as concentrações dos reagentes e produtos são as mesmas.
d) No instante x, a velocidade V2 é máxima.
e) No instante x, as concentrações de A e B são as mesmas que no instante
y.
25) (UFRS)O gás NO, quando liberado na atmosfera, transforma-se num
gás poluente.A reação de formação do NO, em equilíbrio, pode ser
representada por:
O2(g) + N2(g)  2NO(g) H>0 :
Nessa situação:
a) o aumento da concentração de N2 favorece a formação de NO.
b)a contração de volume favorece a formação de NO. I
c) o fornecimento de calor ao sistema aumenta a concentração de N2.
d) o sistema, estando em equilíbrio, não pode ser modificado.
e) a retirada de NO aumenta a concentração de O2,
26) (Fuvest-SP) A25°C, o pH de uma solução aquosa de certo eletrólito é igual a 14. Qual a concentração de OH - dessa solução?
a) 1 mol/L
b) 7 mol/L
c) 14 mol/L
d) 10-7 mol/L
e) 10-14mol/L
27) (UFPI) Dada a afirmação: “A urina é uma solução aquosa que apresenta pH = 5”, podemos concluir que:
a) a solução tem caráter básico.
b) a concentração hidrogeniônica é 10-5 mol/L.
c) a concentração hidroxiliônica é 10-7mol/L.
d) a constante de ionização da água é 10-5.
e) a urina é uma solução não eletrolitica.
28) (PUC-MG) Um suco de limão apresenta pH = 3. A concentração de H + na solução é, em mol/L, igual a:
a) 1,000.
c) 0,001.
e) 0,006.
b) 0,003.
d) 0,008.
29) (UFMG) A tabela mostra o pH característico de alguns sistemas:
Sistema Vinagre Suco de laranja Suco de tomate Saliva Leite
3
4
5
6
6,8
pH
Baseando-se na tabela apresentada, podemos afirmar que:
a) a clara de ovo é o sistema menos ácido.
b) o sangue é o líquido mais próximo da neutralidade.
c) o suco de laranja é 1,5 vez mais ácido que a saliva.
d) o suco de tomate é duas vezes menos ácido que o o vinagre.
e) todos os líquidos da tabela são ácidos.
Sangue
7,4
Clara de ovo
8
Eletroquímica
1) Principalmente para os elementos representativos da classificação periódica ,há uma regra geral. Mas isso não significa que
existam compostos contendo elementos com todas essas faixas de números de oxidação. Tais regras são apenas orientações
interessantes, pois não existem compostos com elementos fora dessas faixas:
O texto permite concluir que:
a) O Nox máximo do nitrogênio(5A) é -3, por exemplo,emHN03.
40
b) O Nox mínimodosódio(1A) é -7, por exemplo,em NaCI.
c) OS valores máximo e mínimo do Nox do enxofre(6A) são, respectivamente+6 e -2, por exemplo em H2S04e H2S.
d) O Nox máximo do oxigênio é -2.
e) O Nox mínimo do cloro é + 7.
2) Ainda com relação ao texto do teste anterior, contêm fósforo (5A) apenas com Nox máximo:
a) PH3 e H3P03
d) H3P04 e H4P207
b) H3P03 e H3P02
e) NaH2P03 e H4P207
c) PH3 e H3P O 4
3) As espécies químicasC12,Cl-, ClO-e ClO-4; apresentam átomos de cloro com Nox respectivamente iguais a:
a) - 1; - 1; - 1; - 1 b) zero; -1; -1; -1 c) zero ;zero;+ 1; +7
d) zero; - 1; + 1; +7
e) -1; zero ;-1;-7
4) Escreva as fórmulas eletrônicas e indique o Nox de cada átomo nos compostos abaixo:
a) CH4(metano)
b) CCI4(tetracloro -metano)
c) CHCI3(clorofórmio)
5) Determine o Nox do nitrogênio nas espécies químicas :
N2,HN03e NO-3;.
6) Identifique um peróxido entre as substâncias com fórmulas K 20,Ca02e OF2.
7) Em qual das estruturas abaixo o carbono apresenta maior número de oxidação?Justifique sua resposta.
8) O dicromato de amônio,(NH4)2Cr207,é um sala laranja do muito utilizado na curtição de couro e na fabricação de fogos de artifício.
Determine o número de oxidação de cada átomo nesse sal.
9)O gás propano, que pode ser encontrado em botijões de cozinha, possui fórmula:
a) Aplicando o modelo das quebras de ligações, descubra o Nox de cada átomo de carbono.
b) Qual é a média aritmética dos valores de Nox encontrados no item anterior?
c) Qual será o Nox de cada carbono se você aplicar as regras práticas na fórmula molecular do propano, C 3H8?
10) Determino e Nox do hidrogênio nas espécies químicas:H 2. HBr e BaH2.
11) A formação da ferrugem pode ser dada pela seguinte equação química não-balanceada:
Fe(s)+ 02(g)+ H20(vapor)  Fe203.3H20 (ferrugem)
A respeito desse processo, assinale a alternativa correta:
a) A formação de ferrugem não necessita da presença de oxigênio do ar.
b) A formação de ferrugem não necessita de vapor de água existente no ar.
c) O metal ferro sofre redução.
d) O gás oxigênio sofre oxidação.
e) O elemento ferro sofre oxidação, passando do Nox= zero para Nox= +3.
12) Sobre a reação da respiração celular: C6H1206+ 602  6C02 + 6H20, há as seguintes afirmativas:
I) O carbono da glicose(C6H12O6) possui Nox=0
41
II)O carbono sofre oxidação.
III) A glicose é o agente oxidante.
Está(ão) correta(s) somente a(s) afirmativa(s):
a) I.
b) II
c) III
d) I e II. e) II e III.
13) Os cientistas acreditam que a combustão do hidrogênio será uma das principais fontes de energia do século XXI. Veja: 2H 2(g) +
02(g)  2H20(g). Nesse processo:
a) H2 não sofre oxirredução.
b) O2 não sofre oxirredução.
c) H2 sofre redução e é o agente oxidante.
d) o oxigênio se reduz de Nox= O para Nox= -2.
e) H20 é o agente redutor.
14) Sobre a oxirredução, pode-se afirmar que:
a) o oxidante e o redutor sempre são produtos da reação.
c) o redutor se reduz.
d) o oxidante doa elétrons.
b) o oxidante se oxida.
e) o redutor perde elétrons.
15) A reação do sódio com o oxigênio pode produzir óxido de sódio.Veja: 4Na + O2 2Na20. Nessa equação:
a) o sódio sofre redução.
b) o gás O2 sofre oxidação.
c) o sódio é agente redutor.
d) o óxido é agente oxidante.
e) não se verifica oxirredução.
16) No processo abaixo, temos a produção de gás hilariante, nome dado em função das contrações dos músculos faciais provocadas
pela inalação do gás.
NH4N03N20 + 2H20
Nesse processo, o elemento nitrogênio:
a) sofre auto-oxirredução, porque passa de (- 3) no NH 4 e (+5) no N0 3 para(+ 1) no N20.
b) não sofre oxirredução.
c) somente sofre oxidação.
d) somente sofre redução.
e) sofre auto-oxirredução, porque passa de (+3) no NH 4 e (-5) no N0 3 para (-1) no N20.
17) Observe às seguintes semi-reações de eletrodos:
Pb2+ + 2e-  Pbo E 0red = -0,13 V
A13++ 3e-  Alo
E 0red = -1,66 V
Em uma pilha com eletrodos de chumbo e alumínio, pode-se prever que:
a) o eletrodo de alumínio é o cátodo.
b) o eletrodo de chumbo é o pólo negativo.
c) a massa da placa de chumbo diminuirá.
d) Eo vale 1,53 V.
e) a concentração de A13+ irá diminuir.
18) Em uma lista de potenciais, o melhor agente oxidante é a espécie que possui maior tendência de sofrer redução Nas semi-reações
abaixo,o melhor agente oxidante será:
Pb2+ + 2e-  Pbo E 0red = -0,13 V
A13++ 3e-  Alo
E 0red = -1,66 V
2+
a) Pb
b) Pbo
c) A13+
d) AIo
e) Pb2+ouA13+
19) Em uma lista de potenciais, o melhor agente oxidante é a espécie que possui maior tendência de sofrer redução Nas semi-reações
abaixo,o melhor agente redutor será:
Pb2+ + 2e-  Pbo E 0red = -0,13 V
A13++ 3e-  Alo
a) Pb2+
E 0red = -1,66 V
b) Pbo
c) A13+
d) AIo
e) Pb2+ouA13+
20) Observe a seguinte lista de potenciais padrão de redução:
Li+ + e- Lio
EO= -3,0 V
+
Na + e  Não
EO= - 2,7 V
2+
o
Zn + 2e  Zn
EO= -0,76 V
+
2H + 2e  H2
E O= 0
2+
o
Cu + 2e  Cu
EO= +0,34 V
I2+ + 2e-  2IEO= +0,53 V
42
Na pilha Na-Cu, pode-se prever que:
a) o eletrodo de sódio é o pólo negativo.
c) a concentração de Cu2+ aumentará.
e) a massa da placa de cobre diminuirá.
b) Eo = 2,36 V.
d) o eletrodo de cobre é o ânodo.
21) Escreva a reação da seguinte pilha:
Zno  Zn2+  Fe3+ Feo
22) Mg  Mg2+ Cu2+ Cu, determine, com relação à pilha
a) o pólo positivo;
b) o ânodo;
c) as semi-reações
d) a reação global.
23) Uma pilha níquel-crômio apresenta a seguinte equação química, ainda não balanceada:
N 2aq + Cr 0s  N 0aq + C 3aq
a) Efetue o ajuste de coeficientes da equação.
b) Determine qual eletrodo será o pólo negativo.
c) Indique as semi-reações
24) As questões abaixo referem-se a uma pilha com as semi-reações:
Ag++ e-  AgO
EO= +0,80 V
2+
O
Mg + 2e  Mg
EO= -2,37 V
a) Escreva as semi-reações de cada eletrodo.
b) Identifique:cátodo, ânodo, pólo negativo e pólo positivo.Justifique cada escolha.
c) Calcule o valor de Eo.
d) Qual é o melhor agente oxidante da pilha?
25) Todos sabemos que objetos de prata ficam escuros. Tal fenômeno ocorre em virtude da oxidação da prata, que forma compostos
como Ag2O ou Ag2S. Se você embrulhar objetos de prata em papel de alumínio e mergulhá-los em solução aquosa de bicarbonato de
sódio, a cor escura da prata irá desaparecer. Uma outra maneira de clarear esses objetos consiste em colocá-los em uma panela de
alumínio contendo uma solução de bicarbonato de sódio.
Proponha uma explicação para esses fenômenos
Dados:
A13++ 3e-  AIo
EO= -1,66 V
+
O
Ag + e  Ag
EO= +0,80 V
25) Na eletrólise ígnea de NiF2 há formação de:
a) Ni2+, no cátodo.
b) F-, no ânodo.
d) gás flúor, no cátodo.
e) NiF2 sólido.
c) Nio, no cátodo.
26) O magnésio é obtido da água do mar. Isola-se o magnésio na forma de MgCI2 e, em seguida, realiza-se uma eletrólise ígnea.
Pode-se afirmar que o magnésio:
a) é produzido no pólo negativo da eletrólise.
b) é obtido no ânodo.
c) é obtido na forma de Mg3+.
d) recebeu apenas um elétron para tornar-se um átomo neutro.
e) é obtido num processo que não é de oxirredução.
27) Sobre a eletrólise aquosa do KCl, veja as seguintes afirmações:
I) Uma solução aquosa de KCl possui apenas íons K +e CI-.
II) K+, metal alcalino, não reage em eletrólises aquosas.
III) O íon Cl- não reagirá.
Está(ão) correta(s) somente a(s) afirmação (ões):
a) I.
b) II.
c) III.
d) I e III.
e) II e III.
28) Defina, da maneira que você sabe, os seguintes termos e expressões:
a) ânodo
b) cátodo
c) cátodo e ânodo da eletrólise
d) como prever reações com valores de E°
43
e)
f)
g)
h)
i)
j)
k)
l)
m)
n)
o)
p)
q)
r)
corrosão de metal .
E°
eletrodo de sacrifício
eletrólise
eletrólise aquosa
eletrólise ígnea
número de oxidação
oxidação
oxidante
pólo negativo
pólo positivo
potencial de eletrodo
redução
redutor
29) (UFPA)Considere a seguinte equação de redox:
Cr20 72 ;- + H2S+ W -+ Cr3++ S + H20
A soma total dos coeficientes das espécies químicas envolvidas, após balanceamento da equação,será iguala:
a) 6.
b) 16.
c) 22.
d) 24
e) 28.
30) (AMAN-RJ) Ajuste,por oxirredução, os coeficientes da reação:
H2S04+ Cu-+ CuS04+ S02+ H20
Somando os coeficientes encontrados ,obtemos:
a) 10.
b)9
c) 6.
d)7
e) 8.
31) (Cesgranrio-RJ) Após o balanceamento da equação:
Mn0 4 + Fe2++ H+  Mn2++ Fe3+ + H20
os coeficientes do Fe2+e do Fe3+serão, respectivamente:
a) 1 e 1. b) 2 e 3. c) 3 e 2. d) 3 e 3. e) 5 e 5.
32) (PUC-MG)Na cela eletroquímica representada pela equação Nio+ 2Ag+ + Ni2+ + 2AgO, é correto afirmar que:
a) os elétrons fluem,pelo circuito externo,da prata para o níquel.
b) o cátodo é o eletrodo de níquel.
c) o eletrodo de prata sofre desgaste.
d) a prata sofre redução .
e) a solução de níquel irá se diluir.
33) (UFPI)Dado um sistema relativo a uma pilha de Mg e Zn:
Sabendo que os elétrons fluem do eletrodo de magnésio para o eletrodo de zinco,podemos
afirmar que:
a) a reação não é espontânea.
b) o eletrodo de magnésio é o pólo positivo.
c) o eletrodo de zinco é o cátodo.
d) o eletrodo de zinco sofre corro são.
e) a concentração de Mg2-diminui.
34) (UFMS)Dadas as semi-reações e os seus respectivos potenciais normais de oxidação:
Fe Fe2++ 2eEO= 0,44 V
2+
Mg  Mg + 2e
EO= 2,38 V
2F  F2(g)+ 2e
EO= -2,87 V
Assinale a(s) alternativa(s) que indique(m) o(s) melhor(es) agente(s) oxidante(s):
a) ferro metálico.
b) magnésio metálico.
c) fluoreto de magnésio. d) fluoreto de ferro II.
e) íons Fe2+.
f) íons Mg2+.
g) flúor gasoso.
35) (UFSC) A partir dos dados fornecidos pelas semi-reações abaixo, escreva a equação química total que represente um processo
espontâneo.
44
Assinale as afirmações corretas com relação à equação química total:
a) Flúor é o agente oxidante.
b) Ferro é o agente redutor.
c) A diferença de potencial é iguala +2,43 V.
d) O ferro oxida em presença de flúor.
e) Não representa uma reação de oxirredução.
36) (UFSC) Dadas as semi-reações e seus respectivos potenciais padrão:
Cu2++ 2e-  Cu°
EO= +0,340 V
2+
Zn + 2e  Zn°
EO= -0,780 V
+
Ag + 1e  Ag°
EO= +0,779 V
Hgh + 2e  Hg°
EO= +0,854 V
Assinale qual(is) das reações abaixo é (são) possível(is), indicando os produtos.
a) Zn2++ Ag
b) Ag++ Zn
+
c) Ag + Cu
d) Cu2++ Zn
2+
e) Zn + Cu
f) Hg2++ Zn
37) (PUC-MG) Dada a célula eletroquímica ilustrada, considere os seguintes
potenciais padrão de redução: EºZn2+/ Znº = -0,76 V;
EºAu3+/Auo = +
1,50V
A partir dessas informações, responda:
a) Qual é a reação catódica?
b) Qual é a reação anódica?
c) Qual é a reação global da célula?
d) Qual é a ddp da pilha?
e) A célula é espontânea?
45
Download